You are on page 1of 130

© 2022 by R. C. Hibbeler. Published by Pearson Education, Inc., Hoboken, NJ. All rights reserved.

This material is protected under all copyright laws


as they currently exist. No portion of this material may be reproduced, in any form or by any means, without permission in writing from the publisher.

2–1.

Determine the magnitude of the resultant force y


FR = F1 + F2 and its orientation u, measured
counterclockwise from the positive x axis.
F1 5 260 lb

12 13

SOLUTION x

Sine Law: 458


sin 67.62° sin (45° + a)
= a = 3.728°
320 260
u = 180° - a = 176° Ans. F 5 310 lb
2

12
tan-1 = 67.38°
5
67.38° + 45° = 112.38°
360° - a (112.38°)
= 67.62°
a

Cosine Law:

FR = 23102 + 2602 - 2(310)(260) cos 67.62° = 320 lb Ans.

Ans:
FR = 320 lb
u = 176°

22
© 2022 by R. C. Hibbeler. Published by Pearson Education, Inc., Hoboken, NJ. All rights reserved. This material is protected under all copyright laws
as they currently exist. No portion of this material may be reproduced, in any form or by any means, without permission in writing from the publisher.

2–2.

Determine the magnitude of the resultant force y


FR′ = F1 - F2 and its orientation u, measured
counterclockwise from the positive x axis.
F1 5 260 lb

12 13

SOLUTION x

12 458
tan-1 = 67.38°
5
67.38° + 45° = 112.38°

F2 5 310 lb
Cosine Law:

FR = 23102 + 2602 - 2(310)(260) cos 112.38° = 474 lb Ans.

Sine Law:
sin 112.38° sin (u - 45°)
= u = 75.4° Ans.
474 260

Ans:
FR = 474 lb
u = 75.4°

23
© 2022 by R. C. Hibbeler. Published by Pearson Education, Inc., Hoboken, NJ. All rights reserved. This material is protected under all copyright laws
as they currently exist. No portion of this material may be reproduced, in any form or by any means, without permission in writing from the publisher.

2–3.

Two forces are applied at the end of a screw eye in order to y


remove the post. Determine the angle u 10° … u … 90°2 F
and the magnitude of force F so that the resultant force
30°
acting on the post is directed vertically upward and has a 500 N θ
magnitude of 750 N.

SOLUTION
Parallelogram Law: The parallelogram law of addition is shown in Fig. (a).

Trigonometry: Using law of sines [Fig. (b)], we have

sin f sin 30°


=
750 500

sin f = 0.750

f = 131.41° 1By observation, f 7 80°2

Thus,

u = 180° - 30° - 131.41° = 18.59° = 18.6° Ans.

F 500
=
sin 18.59° sin 30°

F = 319 N Ans.

Ans:
u = 18.6°
F = 319 N

24
© 2022 by R. C. Hibbeler. Published by Pearson Education, Inc., Hoboken, NJ. All rights reserved. This material is protected under all copyright laws
as they currently exist. No portion of this material may be reproduced, in any form or by any means, without permission in writing from the publisher.

*2–4.
Determine the magnitudes
The vertical force of the
F acts downward at two components
on the of F
two-membered
along
frame.members
Determine and
ABthe AC. Set F of
magnitudes 500two
= the N. components of
B
F directed along the axes of and . Set 500 N.

SOLUTION A
Parallelogram Law:
Parallelogram Law: The parallelogram law of addition is shown in Fig. a.
Trigonometry:
Trigonometry: Using the law of sines (Fig. b), we have
F
500 C
sin 60° sin 75°

448 N Ans.

500
sin 45° sin 75°

366 N Ans.

Ans:
FAB = 448 N
FAC = 366 N

25
© 2022 by R. C. Hibbeler. Published by Pearson Education, Inc., Hoboken, NJ. All rights reserved. This material is protected under all copyright laws
as they currently exist. No portion of this material may be reproduced, in any form or by any means, without permission in writing from the publisher.

2–5.

Solve Prob. 2-4 with F = 350 lb.


B

45

SOLUTION A

Parallelogram Law: The parallelogram law of addition is shown in Fig. a.


Trigonometry: Using the law of sines (Fig. b), we have F 30
FAB 350 C
=
sin 60° sin 75°

FAB = 314 lb Ans.

FAC 350
=
sin 45° sin 75°

FAC = 256 lb Ans.

Ans:
FAB = 314 lb
FAC = 256 lb

26
© 2022 by R. C. Hibbeler. Published by Pearson Education, Inc., Hoboken, NJ. All rights reserved. This material is protected under all copyright laws
as they currently exist. No portion of this material may be reproduced, in any form or by any means, without permission in writing from the publisher.

2–6.

Determine the magnitude of the resultant force v


FR = F1 + F2 and its direction, measured clockwise from
the positive u axis.
30
75
F1  4 kN

30

F2  6 kN
SOLUTION
Parallelogram Law: The parallelogram law of addition is shown in Fig. a,
Trigonometry: Applying Law of cosines by referring to Fig. b,

FR = 242 + 62 - 2(4)(6) cos 105° = 8.026 kN = 8.03 kN Ans.

Using this result to apply Law of sines, Fig. b,


sin u sin 105°
= ; u = 46.22°
6 8.026
Thus, the direction f of FR measured clockwise from the positive u axis is
f = 46.22° - 45° = 1.22° Ans.

Ans:
FR = 8.03 kN
f = 1.22°

27
© 2022 by R. C. Hibbeler. Published by Pearson Education, Inc., Hoboken, NJ. All rights reserved. This material is protected under all copyright laws
as they currently exist. No portion of this material may be reproduced, in any form or by any means, without permission in writing from the publisher.

2–7. v
Resolve the force F1 into components acting along the u
and v axes and determine the magnitudes of the components. 30
75
F1  4 kN

30

F2  6 kN

SOLUTION
Parallelogram Law: The parallelogram law of addition is shown in Fig. a,
Trigonometry: Applying the sines law by referring to Fig. b.
(F1)v 4
= ; (F1)v = 2.928 kN = 2.93 kN Ans.
sin 45° sin 105°
(F1)u 4
= ; (F1)u = 2.071 kN = 2.07 kN Ans.
sin 30° sin 105°

Ans:
(F1)v = 2.93 kN
(F1)u = 2.07 kN

28
© 2022 by R. C. Hibbeler. Published by Pearson Education, Inc., Hoboken, NJ. All rights reserved. This material is protected under all copyright laws
as they currently exist. No portion of this material may be reproduced, in any form or by any means, without permission in writing from the publisher.

*2–8. v

Resolve the force F2 into components along the u and


v axes and determine the magnitudes of the components. 30
75
F1  4 kN

30

F2  6 kN

SOLUTION
Parallelogram Law: The parallelogram law of addition is shown in Fig. a,
Trigonometry: Applying the sines law of referring to Fig. b,
(F2)u 6
= ; (F2)u = 6.00 kN Ans.
sin 75° sin 75°
(F2)v 6
= ; (F2)v = 3.106 kN = 3.11 kN Ans.
sin 30° sin 75°

Ans:
(F2)u = 6.00 kN
(F2)v = 3.11 kN

29
© 2022 by R. C. Hibbeler. Published by Pearson Education, Inc., Hoboken, NJ. All rights reserved. This material is protected under all copyright laws
as they currently exist. No portion of this material may be reproduced, in any form or by any means, without permission in writing from the publisher.

2–9.

Determine the magnitude of the resultant force y


FR = F1 + F2 and its orientation u, measured clockwise
from the positive x axis. F3 5 250 N

308

x
308

SOLUTION 458
F1 5 400 N
Cosine Law: F2 5 360 N
2 2
FR = 2360 + 400 - 2(360)(400) cos 75° = 463.75 = 464 N Ans.

Sine Law:
sin 75° sin (u - 30°)
= u = 78.6° Ans.
463.75 360

Ans:
FR = 464 N
u = 78.6°

30
© 2022 by R. C. Hibbeler. Published by Pearson Education, Inc., Hoboken, NJ. All rights reserved. This material is protected under all copyright laws
as they currently exist. No portion of this material may be reproduced, in any form or by any means, without permission in writing from the publisher.

2–10.

Determine the magnitude of the resultant y


force FR = F1 + F3 and its orientation u, measured
counter­clockwise from the positive x axis. F3 5 250 N

308

x
308

SOLUTION 458
F1 5 400 N
Cosine Law: F2 5 360 N
2 2
FR = 2250 + 400 - 2(250)(400) cos 30° = 222.02 = 222 N Ans.

Sine Law:
sin 30° sin (30° + u)
= u = 4.26° Ans.
222.02 250

Ans:
FR = 222 N
u = 4.26°

31
© 2022 by R. C. Hibbeler. Published by Pearson Education, Inc., Hoboken, NJ. All rights reserved. This material is protected under all copyright laws
as they currently exist. No portion of this material may be reproduced, in any form or by any means, without permission in writing from the publisher.

2–11.
If 60°, determine
u =plate
The the to
is subjected magnitude
the twoofforces
the resultant forceBand
at A and as FA 8 kN
its direction
shown. If u measured clockwise
= 60°, determine from
the the horizontal.
magnitude of the resultant
of these two forces and its direction measured clockwise u
from the horizontal. A

SOLUTION
Parallelogram Law:
Parallelogram Law: The parallelogram law of addition is shown in Fig. a.

Trigonometry:
Trigonometry: Using law of cosines (Fig. b), we have

FR = 282 + 62 - 2(8)(6) cos 100°


40
= 10.80 kN = 10.8 kN Ans.
B
The angle u can be determined using law of sines (Fig. b). FB 6 kN

sin u sin 100°


=
6 10.80
sin u = 0.5470

u = 33.16°

Thus, the direction f of FR measured from the x axis is

f = 33.16° - 30° = 3.16° Ans.

Ans:
FR = 10.8 kN
f = 3.16°

32
© 2022 by R. C. Hibbeler. Published by Pearson Education, Inc., Hoboken, NJ. All rights reserved. This material is protected under all copyright laws
as they currently exist. No portion of this material may be reproduced, in any form or by any means, without permission in writing from the publisher.

*2–12.

Determine the angle of u for connecting member A to the FA 8 kN


plate so that the resultant force of FA and FB is directed
horizontally to the right. Also, what is the magnitude of the u
resultant force? A

SOLUTION
Parallelogram Law:
Parallelogram Law: The parallelogram law of addition is shown in Fig. a.

Trigonometry:
Trigonometry: Using law of sines (Fig .b), we have

sin (90° - u) sin 50°


= 40
6 8
B
sin (90° - u) = 0.5745
FB 6 kN
u = 54.93° = 54.9° Ans.

From the triangle, f = 180° - (90° - 54.93°) - 50° = 94.93°. Thus, using law of
cosines, the magnitude of FR is

FR = 282 + 62 - 2(8)(6) cos 94.93°

= 10.4 kN Ans.

Ans:
u = 54.9°
FR = 10.4 kN

33
© 2022 by R. C. Hibbeler. Published by Pearson Education, Inc., Hoboken, NJ. All rights reserved. This material is protected under all copyright laws
as they currently exist. No portion of this material may be reproduced, in any form or by any means, without permission in writing from the publisher.

2–13.

The pelvis P is connected to the femur F at A using three y


different muscles, which exert the forces shown on the femur.
120 N
Determine the resultant force and specify its orientation u,
measured counterclockwise from the positive x axis. P
13 12
5
60 N
80 N

308
A x

SOLUTION F
Cosine Law:

FR1 = 2602 + 802 - 2(60)(80) cos 120° = 121.66 N

Sine Law:
sin 120° sin f
= f = 25.28°
121.66 60

Cosine Law:

FR = 21202 + 121.662 - 2(120)(121.66) cos 167.90°


= 240.31 = 240 N Ans.

Sine Law:
sin a sin 167.90°
= a = 6.01°
120 240.31
u = 55.28° + a = 61.3° Ans.

Ans:
FR = 240 N
u = 61.3°

34
© 2022 by R. C. Hibbeler. Published by Pearson Education, Inc., Hoboken, NJ. All rights reserved. This material is protected under all copyright laws
as they currently exist. No portion of this material may be reproduced, in any form or by any means, without permission in writing from the publisher.

2–14.

If the resultant force acting on the support is to be 1200 lb, F


directed horizontally to the right, determine the force F in A
rope A and the corresponding angle u.
u

60
900 lb

SOLUTION
Parallelogram Law: The parallelogram law of addition is shown in Fig. a,
Trigonometry: Applying the law of cosines by referring to Fig. b,

F = 29002 + 12002 - 2(900)(1200) cos 30° = 615.94 lb = 616 lb Ans.

Using this result to apply the sines law, Fig. b,


sin u sin 30°
= ; u = 46.94° = 46.9° Ans.
900 615.94

Ans:
F = 616 lb
u = 46.9°

35
© 2022 by R. C. Hibbeler. Published by Pearson Education, Inc., Hoboken, NJ. All rights reserved. This material is protected under all copyright laws
as they currently exist. No portion of this material may be reproduced, in any form or by any means, without permission in writing from the publisher.

2–15.

The plate is subjected to the forces acting on members A y


and B as shown. If u = 60°, determine the magnitude of the FA  400 lb
resultant of these forces and its direction measured
clockwise from the positive x axis.
30 A

SOLUTION x
Cosine
Cosine law:
Law: u
B
FR = 25002 + 4002 - 2(500)(400) cos 60° = 458 lb Ans.

Law:
Sine law: FB  500 lb

sin 60° sin (60°- a)


= a = 10.9° Ans.
458 400

Ans:
FR = 458 lb
a = 10.9°

36
© 2022 by R. C. Hibbeler. Published by Pearson Education, Inc., Hoboken, NJ. All rights reserved. This material is protected under all copyright laws
as they currently exist. No portion of this material may be reproduced, in any form or by any means, without permission in writing from the publisher.

*2–16.

Determine the angle u for connecting member B to the y


plate so that the resultant angle of FA and FB is directed FA  400 lb
along the positive x axis. What is the magnitude of the
resultant force?
30 A

SOLUTION x
Sine law:
Law: u
B
sin u sin 60°
= u = 43.9° Ans.
400 500
FR 500
= FR = 561 lb Ans. FB  500 lb
sin (120°- 43.9°) sin 60°

Ans:
u = 43.9°
FR = 561 lb

37
© 2022 by R. C. Hibbeler. Published by Pearson Education, Inc., Hoboken, NJ. All rights reserved. This material is protected under all copyright laws
as they currently exist. No portion of this material may be reproduced, in any form or by any means, without permission in writing from the publisher.

2–17.

Determine the magnitude and direction of the resultant y


FR = F1 + F2 + F3 of the three forces by first finding the F1 30 N
resultant F¿ = F1 + F2 and then forming
finding FR = F¿ + F3. 5
3
4 F3 50 N
x

20

SOLUTION
F2 20 N

F¿ = 2(20)2 + (30)2 - 2(20)(30) cos 73.13° = 30.85 N

30.85 30
= ; u¿ = 1.47°
sin 73.13° sin (70° - u¿)

FR = 2(30.85)2 + (50)2 - 2(30.85)(50) cos 1.47° = 19.18 = 19.2 N Ans.

19.18 30.85
= ; u = 2.37° Ans.
sin 1.47° sin u

Ans:
FR = 19.2 N
u = 2.37° c

38
© 2022 by R. C. Hibbeler. Published by Pearson Education, Inc., Hoboken, NJ. All rights reserved. This material is protected under all copyright laws
as they currently exist. No portion of this material may be reproduced, in any form or by any means, without permission in writing from the publisher.

2–18.

Determine the magnitude and direction of the resultant y


FR = F1 + F2 + F3 of the three forces by first finding the F1 30 N
resultant F¿ = F2 + F3 and then forming FR = F¿ + F1. 5
3
4 F3 50 N
x

20

SOLUTION
F2 20 N
¿ 2 2
F = 2(20) + (50) - 2(20)(50) cos 70° = 47.07 N

20 47.07
= ; u¿ = 23.53°
sin u ¿ sin 70°

FR = 2(47.07)2 + (30)2 - 2(47.07)(30) cos 13.34° = 19.18 = 19.2 N Ans.

19.18 30
= ; f = 21.15°
sin 13.34° sin f

u = 23.53° - 21.15° = 2.37° Ans.

Ans:
FR = 19.2 N
u = 2.37° c

39
© 2022 by R. C. Hibbeler. Published by Pearson Education, Inc., Hoboken, NJ. All rights reserved. This material is protected under all copyright laws
as they currently exist. No portion of this material may be reproduced, in any form or by any means, without permission in writing from the publisher.

2–19.
The
Theplate
plateis is
subjected to the
subjected to two
the forces at A and
two forces at B. u =B60°,
A Ifand as FA 8 kN
determine
shown. If uthe= magnitude of thethe
60°, determine resultant force of
magnitude andthe
itsresultant
direction
measured fromforces
of these two the positive
and itsx axis.
direction measured clockwise u
from the horizontal. A

SOLUTION
Law: The parallelogram law of addition is shown in Fig. a.
Parallelogram Law:

Trigonometry:
Trigonometry: Using law of cosines (Fig. b), we have

FR = 282 + 62 - 2(8)(6) cos 100°


40
= 10.80 kN = 10.8 kN Ans.
B
The angle u can be determined using law of sines (Fig. b). FB 6 kN

sin u sin 100°


=
6 10.80
sin u = 0.5470

u = 33.16°

Thus, the direction f of FR measured from the x axis is

f = 33.16° - 30° = 3.16° Ans.

Ans:
FR = 10.8 kN
f = 3.16°

40
© 2022 by R. C. Hibbeler. Published by Pearson Education, Inc., Hoboken, NJ. All rights reserved. This material is protected under all copyright laws
as they currently exist. No portion of this material may be reproduced, in any form or by any means, without permission in writing from the publisher.

*2–20.
Determine
Determine thethe angle
angleofu ufor
forconnecting
connectingmember
member A A to the
to the FA 8 kN
plate
plate so
so that
thatthe
theresultant
resultantofforce
FA and
of FFBAisand
directed
FB isalong the
directed
positive x axis.
horizontally to Also, whatAlso,
the right. is the magnitude
what of the resultant
is the magnitude of the u
force?
resultant force? A

SOLUTION
Parallelogram Law: The parallelogram law of addition is shown in Fig. a.

Trigonometry: Using law of sines (Fig .b), we have

sin (90° - u) sin 50°


= 40
6 8
B
sin (90° - u) = 0.5745
FB 6 kN
u = 54.93° = 54.9° Ans.

From the triangle, f = 180° - (90° - 54.93°) - 50° = 94.93°. Thus, using law of
cosines, the magnitude of FR is

FR = 282 + 62 - 2(8)(6) cos 94.93°

= 10.4 kN Ans.

Ans:
u = 54.9°
FR = 10.4 kN

41
© 2022 by R. C. Hibbeler. Published by Pearson Education, Inc., Hoboken, NJ. All rights reserved. This material is protected under all copyright laws
as they currently exist. No portion of this material may be reproduced, in any form or by any means, without permission in writing from the publisher.

2–21.

angle uu 10°
Determine the design angle (0° … u … 90°)90° 2 for
for member
member A
AB so that the 400-lb
400-lb horizontal
horizontal force
force has
has aa component
component of
of 400 lb
θ
500 lb directed from A toward C. What
What is
is the
the component
component of
of
force acting along
along member
member AB?AB?Take
Takeff == 40°.
40°. φ
B

SOLUTION
Sine law:
Law: C

sin u sin 40°


= u = 53.5° Ans.
500 400

FAB 400
= FAB = 621 lb Ans.
sin (180° - 40° - 53.5°) sin 40°

Ans:
u = 53.5°
FAB = 621 lb

42
© 2022 by R. C. Hibbeler. Published by Pearson Education, Inc., Hoboken, NJ. All rights reserved. This material is protected under all copyright laws
as they currently exist. No portion of this material may be reproduced, in any form or by any means, without permission in writing from the publisher.

2–22.
Determine thethe design angle ff10°
design angle 90° 2 between
(0° …… f … 90°) between 400 lb A A
400 lb
members andand
struts ABAB ACAC so that
so that thethe 400-lb
400-lb horizontal
horizontal force
force hashas
a u u
acomponent
component of of
600600 lb which
lb which actsacts upthe
up to to left,
the right,
in the in the
same
direction from B toward
as from A. Also
B towards calculate
A. Take the
u = 30° . magnitude of f
B f B
the force component along AC. Take u = 30°.

SOLUTION
Parallelogram Law: The parallelogram law of addition is shown in Fig. a.
C
Trigonometry: Using law of cosines (Fig. b), we have
C
FAC = 24002 + 6002 - 2(400)(600) cos 30° = 322.97 lb

The angle f can be determined using law of sines (Fig. b).

sin f sin 30°


=
400 322.97

sin f = 0.6193

f = 38.3° Ans.

Ans:
FAC = 323 lb
f = 38.3°

43
© 2022 by R. C. Hibbeler. Published by Pearson Education, Inc., Hoboken, NJ. All rights reserved. This material is protected under all copyright laws
as they currently exist. No portion of this material may be reproduced, in any form or by any means, without permission in writing from the publisher.

2–23.

Determine the magnitude of the two towing forces FB and


FC if the resultant force has a magnitude FR = 10 kN and is C
directed along the positive x axis. Set u = 15°.
FC

208
x
A u
FB

SOLUTION B

Sine Law:

FC 10
= FC = 4.51 kN Ans.
sin 15° sin 145°

FB 10
= FB = 5.96 kN Ans.
sin 20° sin 145°

Ans:
FC = 4.51 kN
FB = 5.96 kN

44
© 2022 by R. C. Hibbeler. Published by Pearson Education, Inc., Hoboken, NJ. All rights reserved. This material is protected under all copyright laws
as they currently exist. No portion of this material may be reproduced, in any form or by any means, without permission in writing from the publisher.

*2–24.

If the resultant FR of the two forces acting on the jet aircraft


is to be directed along the positive x axis and have a C
magnitude of 10 kN, determine the angle u of the cable
attached to the truck at B so that FB is a minimum. What is FC
the magnitude of force in each cable when this occurs?
208
x
A u
FB

SOLUTION B

u = 90° - 20° = 70° Ans.

FB = 10 sin 20° = 3.42 kN Ans.

FC = 10 cos 20° = 9.40 kN Ans.

Ans:
u = 70°
FB = 3.42 kN
FC = 9.40 kN

45
© 2022 by R. C. Hibbeler. Published by Pearson Education, Inc., Hoboken, NJ. All rights reserved. This material is protected under all copyright laws
as they currently exist. No portion of this material may be reproduced, in any form or by any means, without permission in writing from the publisher.

2–25.

The 500-lb force acting on the frame is to be resolved into F = 500 lb


two components acting along the axis of the struts AB and θ
A
AC. If the component of force along AC is required to be
300 lb, directed from A to C, determine the magnitude of
force acting along AB and the angle u of the 500-lb force.

60°
SOLUTION 45°
B C
Parallelogram Law: The parallelogram law of addition is shown in Fig. (a).

Trigonometry: Using law of sines [Fig. (b)], we have

sin f sin 75°


=
300 500

sin f = 0.5796

f = 35.42° Ans.

Thus,

45° + u + 75° + 35.42° = 180°

u = 24.58° = 24.6°

FAB 500
=
sin145° + 24.58°2 sin 75°

FAB = 485 lb Ans.

Ans:
f = 35.42°
FAB = 485 lb

46
© 2022 by R. C. Hibbeler. Published by Pearson Education, Inc., Hoboken, NJ. All rights reserved. This material is protected under all copyright laws
as they currently exist. No portion of this material may be reproduced, in any form or by any means, without permission in writing from the publisher.

2–26.

Determine the magnitude and direction u of FA so that the y


resultant force is directed along the positive x axis and has a
magnitude of 1250 N.
FA
A
θ

x
O 30°

B
SOLUTION FB = 800 N

+ FR = ΣFx;
S FRx = FA sin u + 800 cos 30° = 1250
x

+ c FRy = ΣFy; FRy = FA cos u - 800 sin 30° = 0

u = 54.3° Ans.

FA = 686 N Ans.

Ans:
u = 54.3°
FA = 686 N

47
© 2022 by R. C. Hibbeler. Published by Pearson Education, Inc., Hoboken, NJ. All rights reserved. This material is protected under all copyright laws
as they currently exist. No portion of this material may be reproduced, in any form or by any means, without permission in writing from the publisher.

2–27.
Determine
Determine the themagnitude
magnitudeof the
andresultant force measured
direction, acting on y
the ring at O, if F = 750 N and u = 45°. What
counterclockwise from the positive x axis, of the resultant
A is its
direction,
force actingmeasured
on the ringcounterclockwise
at O, if FA = 750from
N andthe
u = positive
45°.
x axis? FA
A
θ

x
O 30°

B
SOLUTION FB = 800 N

Scalar Notation: Suming the force components algebraically, we have


+ FR = ΣFx;
S FRx = 750 sin 45° + 800 cos 30°
x

= 1223.15 N S

+ c FRy = ΣFy; FRy = 750 cos 45° - 800 sin 30°

= 130.33 N c

The magnitude of the resultant force FR is

FR = 3F 2Rx + F 2Ry

= 21223.152 + 130.332 = 1230 N = 1.23 kN Ans.

The directional angle u measured counterclockwise from positive x axis is

FRy 130.33
u = tan-1 = tan-1 a b = 6.08° Ans.
FRx 1223.15

Ans:
FR = 1.23 kN
u = 6.08°

48
© 2022 by R. C. Hibbeler. Published by Pearson Education, Inc., Hoboken, NJ. All rights reserved. This material is protected under all copyright laws
as they currently exist. No portion of this material may be reproduced, in any form or by any means, without permission in writing from the publisher.

*2–28.

If F1 = F2 = 30 lb, determine the angles u and f so that the F1


y x
resultant force is directed along the positive x axis and has a
magnitude of FR = 20 lb.
θ

φ F2

SOLUTION
30 30
=
sin f sin u

f = u

(30)2 = (30)2 + (20)2 - 2(30)(20) cos u

f = u = 70.5° Ans.

Ans:
u = f = 70.5°

49
© 2022 by R. C. Hibbeler. Published by Pearson Education, Inc., Hoboken, NJ. All rights reserved. This material is protected under all copyright laws
as they currently exist. No portion of this material may be reproduced, in any form or by any means, without permission in writing from the publisher.

2–29.

If the resultant force of the two tugboats is 3 kN, directed y


along the positive x axis, determine the required magnitude A
of force FB and its direction u.
FA 2 kN

30
x
u
C FB
SOLUTION
The parallelogram law of addition and the triangular rule are shown in Figs. a and b,
respectively.
B
Applying the law of cosines to Fig. b,

FB = 222 + 32 - 2(2)(3)cos 30°

= 1.615kN = 1.61 kN Ans.

Using this result and applying the law of sines to Fig. b, yields

sin u sin 30°


= u = 38.3° Ans.
2 1.615

Ans:
FB = 1.61 kN
u = 38.3°

50
© 2022 by R. C. Hibbeler. Published by Pearson Education, Inc., Hoboken, NJ. All rights reserved. This material is protected under all copyright laws
as they currently exist. No portion of this material may be reproduced, in any form or by any means, without permission in writing from the publisher.

2–30.

If FB = 3 kN and u = 45°, determine the magnitude of the y


resultant force of the two tugboats and its direction A
measured clockwise from the positive x axis.
FA 2 kN

30
x
u
C
SOLUTION FB

The parallelogram law of addition and the triangular rule are shown in Figs. a and b,
respectively.
B
Applying the law of cosines to Fig. b,

FR = 222 + 32 - 2(2)(3) cos 105°

= 4.013 kN = 4.01 kN Ans.

Using this result and applying the law of sines to Fig. b, yields

sin a sin 105°


= a = 46.22°
3 4.013

Thus, the direction angle f of FR, measured clockwise from the positive x axis, is

f = a - 30° = 46.22° - 30° = 16.2° Ans.

Ans:
FR = 4.01 kN
f = 16.2°

51
© 2022 by R. C. Hibbeler. Published by Pearson Education, Inc., Hoboken, NJ. All rights reserved. This material is protected under all copyright laws
as they currently exist. No portion of this material may be reproduced, in any form or by any means, without permission in writing from the publisher.

2–31.

If the resultant force of the two tugboats is required to be


y
directed towards the positive x axis, and FB is to be a A
minimum, determine the magnitude of FR and FB and the
angle u.
FA 2 kN

30
x
u
SOLUTION C FB

For FB to be minimum, it has to be directed perpendicular to FR. Thus,

u = 90° Ans. B

The parallelogram law of addition and triangular rule are shown in Figs. a and b,
respectively.

By applying simple trigonometry to Fig. b,

FB = 2 sin 30° = 1 kN Ans.

FR = 2 cos 30° = 1.73 kN Ans.

Ans:
u = 90°
FB = 1 kN
FR = 1.73 kN

52
© 2022 by R. C. Hibbeler. Published by Pearson Education, Inc., Hoboken, NJ. All rights reserved. This material is protected under all copyright laws
as they currently exist. No portion of this material may be reproduced, in any form or by any means, without permission in writing from the publisher.

*2–32.

The three cable forces act on the eyebolt. Determine two y


possible magnitudes for P so that the resultant force has a
1000 lb
magnitude of 800 N.

4 5

P
x
SOLUTION
308
3
+ FRx = ΣFx;  FRx = P - 400 cos 30° - 1000 a b = P - 946.41
S 5
400 lb
4
+ c FRy = ΣFy;  FRy = 1000 a b - 400 sin 30° = 600
5
FR = 2(P - 946.41)2 + (600)2 = 800

(P - 946.41)2 + (600)2 = (800)2

(P - 946.41)2 = 280 000

P - 946.41 = {529.15

P - 946.41 = 529.15;  P = 1475.6 lb = 1.48 kip  Ans.

P - 946.41 = -529.15;  P = 417 lb  Ans.

Ans:
P = 1.48 kip
P = 417 lb

53
© 2022 by R. C. Hibbeler. Published by Pearson Education, Inc., Hoboken, NJ. All rights reserved. This material is protected under all copyright laws
as they currently exist. No portion of this material may be reproduced, in any form or by any means, without permission in writing from the publisher.

2–33.

Four concurrent forces act on the plate. Determine the y


magnitude of the resultant force and its direction measured
counterclockwise from the positive x axis.
F2 5 100 lb

x
SOLUTION 308 F1 5 60 lb
4
+ FRx = ΣFx;
  FRx = 60 - 38 cos 30° + 5 (50) = 67.09 lb
5
S 3
F3 5 38 lb 4
F4 5 50 lb
3
+ c FRy = ΣFy;  FRy = 100 - 38 sin 30° - (50) = 51 lb
5
FR = 2(67.09)2 + (51)2 = 84.3 lb Ans.

51
u = tan-1 a b = 37.2°  Ans.
67.09

Ans:
FR = 84.3 lb
u = 37.2°

54
© 2022 by R. C. Hibbeler. Published by Pearson Education, Inc., Hoboken, NJ. All rights reserved. This material is protected under all copyright laws
as they currently exist. No portion of this material may be reproduced, in any form or by any means, without permission in writing from the publisher.

2–34.

Express F1 and F2 as Cartesian vectors. y

F1 5 40 lb

308

SOLUTION x
208
Sine Law:

F1 = {40 cos 60°i + 40 sin 60°j} lb = {20i + 34.6j} lb Ans. F2 5 50 lb

F2 = {50 cos 20°i - 50 sin 20°j} lb = {47.0i - 17.1j} lb Ans.

Ans:
F1 = {20i + 34.6j} lb
F2 = {47.0i - 17.1j} lb

55
© 2022 by R. C. Hibbeler. Published by Pearson Education, Inc., Hoboken, NJ. All rights reserved. This material is protected under all copyright laws
as they currently exist. No portion of this material may be reproduced, in any form or by any means, without permission in writing from the publisher.

2–35.

Determine the magnitude of the resultant force and its y


direction measured counterclockwise from the positive
x axis. F1 5 40 lb

308

If F1 = 300 N and u = 10°, determine the magnitude and y


direction, measured counterclockwise from the positive
x¿ axis, of the resultant force acting on the bracket. F2  200 N x
SOLUTION 208
Scalar Analysis:
x¿ F2 5 50 lb
+ FRx = ΣFx;  FRx = 40 cos 60° + 50 cos 20°
S F3  180 N
= 66.98 lb S 60 u F1
SOLUTION 5
13

+ c FRy = ΣFy;  FRy = 40 sin 60° - 50 sin 20° 12

:
+ F 12 x
Rx = ©Fx ; FRx = 300 sin 70° - (180) = 115.8 N
17.54 lb c
=13

Magnitude 5
+ c FRy = ©Fy ; FRy = 300 cos 70° + 200 + (180) = 371.8 N
13
FR = 2FRx2 + FRy2 = 266.982 + 17.542 = 69.2 lb Ans.
FR = 2(115.8)2 + (371.8)2 = 389 N Ans.
Direction
F
B - 1 R ==72.71°
- 1 Ry 17.54
1 371.8
u = tanf = -tan
= tan 14.7° au  Ans.
FRx 66.98
115.8

f¿ = 72.71° - 30° = 42.7° Ans.

Ans:
FR = 69.2 lb
u = 14.7° au

56
© 2022 by R. C. Hibbeler. Published by Pearson Education, Inc., Hoboken, NJ. All rights reserved. This material is protected under all copyright laws
as they currently exist. No portion of this material may be reproduced, in any form or by any means, without permission in writing from the publisher.

*2–36.

Resolve each force acting on the gusset plate into its x and y
y components, and express each force as a Cartesian vector. F3 650 N
3
5 F2 750 N
4

45

x
F1 900 N
F1 = {900(+i)} = {900i} N Ans.

F2 = {750 cos 45°(+ i) + 750 sin 45°(+j)} N


= {530i + 530j} N Ans.

4 3
F3 = e 650a b (+i) + 650 a b (-j) f N
5 5
= {520 i - 390j)} N Ans.

Ans:
F1 = 5900i6 N
F2 = 5530i + 530j 6 N
F3 = 5520i - 390j 6 N

57
© 2022 by R. C. Hibbeler. Published by Pearson Education, Inc., Hoboken, NJ. All rights reserved. This material is protected under all copyright laws
as they currently exist. No portion of this material may be reproduced, in any form or by any means, without permission in writing from the publisher.

2–37.
Determine
Determine thethe magnitude
magnitude of of the
the resultant
resultant force
force acting
acting on
on y
the
the gusset
plate and itsand
plate its direction,
direction, measured
measured counterclockwise
counterclockwise from F3 650 N
from the positive
the positive x axis.x axis. 3
5 F2 750 N
4

SOLUTION
45
Rectangular Components: By referring to Fig. a, the x and y components of F1, F2,
x
and F3 can be written as F1 900 N
(F1)x = 900 N (F1)y = 0
(F2)x = 750 cos 45° = 530.33 N (F2)y = 750 sin 45° = 530.33 N
4 3
(F3)x = 650 a b = 520 N (F3)y = 650a b = 390 N
5 5
Resultant
Resultant Force: Summing the force components algebraically along the x and
y axes, we have

: ©(FR)x = ©Fx;
+ (FR)x = 900 + 530.33 + 520 = 1950.33 N :

+ c ©(FR)y = ©Fy; (FR)y = 530.33 - 390 = 140.33 N c


The magnitude of the resultant force FR is

FR = 2(FR)x2 + (FR)y2 = 21950.332 + 140.332 = 1955 N = 1.96 kN Ans.

The direction angle u of FR, measured clockwise from the positive x axis, is
(FR)y 140.33
u = tan-1 c d = tan-1 a b = 4.12° Ans.
(FR)x 1950.33

Ans:
FR = 1.96 kN
u = 4.12°

58
© 2022 by R. C. Hibbeler. Published by Pearson Education, Inc., Hoboken, NJ. All rights reserved. This material is protected under all copyright laws
as they currently exist. No portion of this material may be reproduced, in any form or by any means, without permission in writing from the publisher.

2–38.

The three forces are applied to the bracket. Determine the 800 N
range of values for the magnitude of force P so that the 3000 N 90
resultant of the three forces does not exceed 2400 N.
60
P

SOLUTION
: FRx = ©Fx; FRx = P + 800 cos 60° - 3000 cos 30° = P - 2198.08

+ c FRy = ©Fy; FRy = 800 sin 60° + 3000 sin 30° = 2192.82

FR = 2(P - 2198.08)2 + (2192.82)2 … 2400

(P - 2198.08)2 + (2192.82)2 … (2400)2

|(P - 2198.08)| … 975.47

- 975.47 … P - 2198.08 … 975.47

1222.6 N … P … 3173.5 N

Ans:
1222.6 N … P … 3173.5 N

59
© 2022 by R. C. Hibbeler. Published by Pearson Education, Inc., Hoboken, NJ. All rights reserved. This material is protected under all copyright laws
as they currently exist. No portion of this material may be reproduced, in any form or by any means, without permission in writing from the publisher.

2–39.

Determine the x and y components of F1 and F2. y

45
F1  200 N

30
SOLUTION
F1x = 200 sin 45° = 141 N Ans. F2  150 N
x
F1y = 200 cos 45° = 141 N Ans.

F2x = - 150 cos 30° = - 130 N Ans.

F2y = 150 sin 30° = 75 N Ans.

Ans:
F1x = 141 N
F1y = 141 N
F2x = - 130 N
F2y = 75 N

60
© 2022 by R. C. Hibbeler. Published by Pearson Education, Inc., Hoboken, NJ. All rights reserved. This material is protected under all copyright laws
as they currently exist. No portion of this material may be reproduced, in any form or by any means, without permission in writing from the publisher.

*2–40.

Determine the magnitude of the resultant force and its y


direction, measured counterclockwise from the positive
x axis.
45
F1  200 N

30
SOLUTION
+R FRx = ©Fx; FRx = - 150 cos 30° + 200 sin 45° = 11.518 N F2  150 N
x
Q +FRy = ©Fy; FRy = 150 sin 30° + 200 cos 45° = 216.421 N

FR = 2 (11.518)2 + (216.421)2 = 217 N Ans.

u = tan - 1 ¢ ≤ = 87.0°
216.421
Ans.
11.518

Ans:
FR = 217 N
u = 87.0°

61
© 2022 by R. C. Hibbeler. Published by Pearson Education, Inc., Hoboken, NJ. All rights reserved. This material is protected under all copyright laws
as they currently exist. No portion of this material may be reproduced, in any form or by any means, without permission in writing from the publisher.

2–41. y
F3  8 kN
Determine the magnitude of the resultant force and its
direction, measured counterclockwise from the positive F2  5 kN
x axis.

60

45
F1  4 kN
x

SOLUTION
Scalar Notation: Summing the force components along x and y axes algebraically by
referring to Fig. a,
S+ (F ) = ΣF ;  (F ) = 4 + 5 cos 45° - 8 sin 15° = 5.465 kN S
R x x R x

+ c (FR)y = ΣFy;  (FR)y = 5 sin 45° + 8 cos 15° = 11.263 kN c

By referring to Fig. b, the magnitude of the resultant force FR is

FR = 2(FR)2x + (FR)2y = 25.4652 + 11.2632 = 12.52 kN = 12.5 kN Ans.

And the directional angle u of FR measured counterclockwise from the positive


x axis is
(FR)y 11.263
u = tan - 1 c d = tan - 1 a b = 64.12° = 64.1° Ans.
(FR)x 5.465

Ans:
FR = 12.5 kN
u = 64.1°

62
© 2022 by R. C. Hibbeler. Published by Pearson Education, Inc., Hoboken, NJ. All rights reserved. This material is protected under all copyright laws
as they currently exist. No portion of this material may be reproduced, in any form or by any means, without permission in writing from the publisher.

2–42.

Express F1, F2, and F3 as Cartesian vectors. y

F3  750 N
45

x
5
3
4

SOLUTION 30 F1  850 N


F2  625 N
4 3
F1 = (850) i - (850) j
5 5
= {680 i - 510 j} N Ans.

F2 = - 625 sin 30° i - 625 cos 30° j

= { -312 i - 541 j} N Ans.

F3 = - 750 sin 45° i + 750 cos 45° j

{ -530 i + 530 j} N Ans.

Ans:
F1 = {680i - 510j} N
F2 = { - 312i - 541j} N
F3 = { - 530i + 530j} N

63
© 2022 by R. C. Hibbeler. Published by Pearson Education, Inc., Hoboken, NJ. All rights reserved. This material is protected under all copyright laws
as they currently exist. No portion of this material may be reproduced, in any form or by any means, without permission in writing from the publisher.

2–43.

Determine the magnitude of the resultant force and its y


direction measured counterclockwise from the positive x axis.
F3  750 N
45

x
5
3
4

SOLUTION 30 F1  850 N


F2  625 N
:
+ F = ©F ; 4
Rx x FRx = (850) - 625 sin 30° - 750 sin 45° = - 162.83 N
5
3
+ c FRy = ©Fy ; FRy = - (850) - 625 cos 30° + 750 cos 45° = -520.94 N
5

FR = 2( - 162.83)2 + ( - 520.94)2 = 546 N Ans.

520.94
f = tan-1 a b = 72.64°
162.83
u = 180° + 72.64° = 253° Ans.

Ans:
FR = 546 N
u = 253°

64
© 2022 by R. C. Hibbeler. Published by Pearson Education, Inc., Hoboken, NJ. All rights reserved. This material is protected under all copyright laws
as they currently exist. No portion of this material may be reproduced, in any form or by any means, without permission in writing from the publisher.

*2–44.

The three
threeconcurrent
concurrentforces
forces acting
acting on on
the the
postpost produce
produce a zeroa y
resultant force FR = 0. If F2 = 12 F1, and F1 is to be 90°
from F2 as shown, determine the required magnitude of F3
expressed in terms of F1 and the angle u.
F2
u
F3
x
SOLUTION
©FRx¿ = 0; F3 cos (u - 90°) = F1

©FRy¿ = 0; F3 sin (u - 90°) = F2 F1

F2 1
tan (u - 90°) = =
F1 2

u - 90° = 26.57°

u = 116.57° = 117° Ans.

F1
F3 =
cos(116.57° - 90°)

F3 = 1.12 F1 Ans.

Ans:
u = 117°
F3 = 1.12 F1

65
© 2022 by R. C. Hibbeler. Published by Pearson Education, Inc., Hoboken, NJ. All rights reserved. This material is protected under all copyright laws
as they currently exist. No portion of this material may be reproduced, in any form or by any means, without permission in writing from the publisher.

2–45.

Three forces act on the ring. Determine the range of values 1500 N
for the magnitude of P so that the magnitude of the
resultant force does not exceed 2500 N. Force P is always
directed to the right.
600 N
60

45
P
SOLUTION
:
+ F = ©F ;
Rx x FRx = P + 600 cos 45° + 1500 cos 105° = P + 36.0355

+ c FRy = ©Fy ; FRy = 600 sin 45° + 1500 sin 105° = 1873.153

(2500)2 = (P + 36.0355)2 + (1873.153)2

; 1655.687 = P + 36.0355

Choose the positive root,

thus

P = 1.62 kN

0 … P … 1.62 kN Ans.

Ans:
0 … P … 1.62 kN

66
© 2022 by R. C. Hibbeler. Published by Pearson Education, Inc., Hoboken, NJ. All rights reserved. This material is protected under all copyright laws
as they currently exist. No portion of this material may be reproduced, in any form or by any means, without permission in writing from the publisher.

2–46.

Determine the magnitude and orientation u of FB so that y


the resultant force is directed along the positive y axis and
FB
has a magnitude of 1500 N. FA  700 N

30 A
B

u
x

SOLUTION
Scalar Notation: Suming the force components algebraically, we have
+ F = ΣF ;
S 0 = 700 sin 30° - FB cos u
Rz x

FB cos u = 350 (1)

+ c FRy = ΣFy; 1500 = 700 cos 30° + FB sin u

FB sin u = 893.8 (2)

Solving Eq. (1) and (2) yields

u = 68.6° FB = 960 N Ans.

Ans:
u = 68.6°
FB = 960 N

67
© 2022 by R. C. Hibbeler. Published by Pearson Education, Inc., Hoboken, NJ. All rights reserved. This material is protected under all copyright laws
as they currently exist. No portion of this material may be reproduced, in any form or by any means, without permission in writing from the publisher.

2–47.
If FB = 600 N and u = 20°, determine the magni­tude of the y
Determine the magnitude and orientation, measured
resultant force and its direction measured counterclockwise
counterclockwise from the positive y axis, of the resultant
from the positive y axis. FB
force acting on the bracket, if FB = 600 N and u = 20°. FA  700 N

30 A
B

u
x

SOLUTION
Scalar Notation: Suming the force components algebraically, we have

+ F = ΣF ;
S FRx = 700 sin 30° - 600 cos 20°
Rx x

= -213.8 N = 213.8 N d

+ c FRy = ΣFy; FRy = 700 cos 30° + 600 sin 20°

= 811.4 N c

The magnitude of the resultant force FR is

FR = 2F 2Rx + F 2Ry = 2213.82 + 811.42 = 839 N Ans.

The directional angle u measured counterclockwise from positive y axis is

FRx 213.8
u = tan-1 = tan-1 a b = 14.8° Ans.
FRy 811.4

Ans:
FR = 839 N
u = 14.8°

68
© 2022 by R. C. Hibbeler. Published by Pearson Education, Inc., Hoboken, NJ. All rights reserved. This material is protected under all copyright laws
as they currently exist. No portion of this material may be reproduced, in any form or by any means, without permission in writing from the publisher.

*2–48.

Three forces act on the bracket. Determine the y


magnitude and direction u of F1 so that the resultant
force is directed along the positive x¿ axis and has a F2  200 N
magnitude of 800 N.

x¿
F3  180 N
60 u F1
SOLUTION 5
13

12

:
+ F = ©F ; 12 x
Rx x 800 sin 60° = F1 sin(60° + u) - (180)
13

5
+ c FRy = ©Fy ; 800 cos 60° = F1 cos(60° + u) + 200 + (180)
13

60° + u = 81.34°

u = 21.3° Ans.

F1 = 869 N Ans.

Ans:
u = 21.3°
F1 = 869 N

69
© 2022 by R. C. Hibbeler. Published by Pearson Education, Inc., Hoboken, NJ. All rights reserved. This material is protected under all copyright laws
as they currently exist. No portion of this material may be reproduced, in any form or by any means, without permission in writing from the publisher.

2–49.

If F = 300 N and
F11 = and uu == 10°,
10°, determine
determinethe
themagnitude
magnitudeofand
the y
resultant force
direction, and itscounterclockwise
measured direction measuredfrom
counterclockwise
the positive
from
x¿ theofpositive
axis, x′ axis.force acting on the bracket.
the resultant F2  200 N

x¿
F3  180 N
60 u F1
SOLUTION 5
13

12

:
+ F = ©Fx ; 12 x
Rx FRx = 300 sin 70° - (180) = 115.8 N
13

5
+ c FRy = ©Fy ; FRy = 300 cos 70° + 200 + (180) = 371.8 N
13

FR = 2(115.8)2 + (371.8)2 = 389 N Ans.

f = tan - 1 B R = 72.71°
371.8
au
115.8

f¿ = 72.71° - 30° = 42.7° Ans.

Ans:
FR = 389 N
f′ = 42.7°

70
© 2022 by R. C. Hibbeler. Published by Pearson Education, Inc., Hoboken, NJ. All rights reserved. This material is protected under all copyright laws
as they currently exist. No portion of this material may be reproduced, in any form or by any means, without permission in writing from the publisher.

2–50.

Express F1 and F2 as Cartesian vectors. y


F1 5 60 lb

F2 5 50 lb

308
208
x

SOLUTION
F1 = - 60 cos 30° i + 60 sin 30° j
= { -52.0 i + 30.0 j} lb Ans.

F2 = - 50 cos 20° i - 50 sin 20° j

  = { - 47.0 i - 17.1 j} lb Ans.

Ans:
F1 = { - 52.0 i + 30.0 j} lb
F2 = { - 47.0 i - 17.1 j} lb

71
© 2022 by R. C. Hibbeler. Published by Pearson Education, Inc., Hoboken, NJ. All rights reserved. This material is protected under all copyright laws
as they currently exist. No portion of this material may be reproduced, in any form or by any means, without permission in writing from the publisher.

2–51.

Determine the magnitude of the resultant force and its y


direction measured counterclockwise from the positive x axis. F1 5 60 lb

F2 5 50 lb

308
208
x

SOLUTION
F1 = - 60 cos 30° i + 60 sin 30° j
= { -52.0 i + 30.0 j} lb

F2 = - 50 cos 20° i - 50 sin 20° j

  = { - 47.0 i - 17.1 j} lb


FR = F1 + F2 = { - 98.9 i + 12.9 j} lb

FR = 2( -98.9)2 + (12.9)2 = 99.8 lb Ans.

12.9
u′ = tan - 1 a b = 7.43°
98.9
  u = 180° - 7.43° = 173°  Ans.

Ans:
FR = 99.8 lb
u = 173°

72
© 2022 by R. C. Hibbeler. Published by Pearson Education, Inc., Hoboken, NJ. All rights reserved. This material is protected under all copyright laws
as they currently exist. No portion of this material may be reproduced, in any form or by any means, without permission in writing from the publisher.

*2–52.

The four concentric forces act on the post. Determine the y


resultant force and its direction, measured counterclockwise
from the positive x axis.
F2 5 600 N

F1 5 300 N

308
SOLUTION x
Scalar Notation: Summing the force components along x and y axes algebraically
by referring to Fig. a,
S+ (F ) = ΣF ;  (F ) = 300 cos 30° + 450 a 5 b - 250 sin 60° 608 13
R x x R x 12
13 F4 5 250 N
5
= 216.38 N S
12
+ c (FR)y = ΣFy;  (FR)y = 300 sin 30° + 600 - 450 a b - 250 cos 60° F3 5 450 N
13
= 209.62 N c

By referring to Fig. b, the magnitude of the resultant force FR is

FR = 2(FR)2x + (FR)2y = 2216.382 + 209.622 = 301.26 N = 301 N Ans.

And the directional angle u of FR measured counterclockwise from the positive


x axis is
(FR)y 209.62
u = tan - 1 c d = tan - 1 a b = 44.09° = 44.1° Ans.
(FR)x 216.38

Ans:
FR = 301 N
u = 44.1°

73
© 2022 by R. C. Hibbeler. Published by Pearson Education, Inc., Hoboken, NJ. All rights reserved. This material is protected under all copyright laws
as they currently exist. No portion of this material may be reproduced, in any form or by any means, without permission in writing from the publisher.

2–53.

Three forces act on the bracket. Determine the magnitude and y


direction u of F2 so that the resultant force is directed along
the positive u axis and has a magnitude of 50 lb.
F3  52 lb

13 12

SOLUTION 5

Scalar Notation: Suming the force components algebraically, we have


F1  80 lb
x
:
5
+ F = ©F ;
Rx x 50 cos 25° = 80 + 52a b + F2 cos (25° + u)
13 25

F2 cos (25° + u) = - 54.684 [1] u


u
12 F2
+ c FRy = ©Fy; -50 sin 25° = 52 a b - F2 sin (25° + u)
13

F2 sin (25° + u) = 69.131 [2]

Solving Eq.[1] and [2] yields

25° + u = 128.35° u = 103° Ans.

F2 = 88.1 lb Ans.

Ans:
u = 103°
F2 = 88.1 lb

74
© 2022 by R. C. Hibbeler. Published by Pearson Education, Inc., Hoboken, NJ. All rights reserved. This material is protected under all copyright laws
as they currently exist. No portion of this material may be reproduced, in any form or by any means, without permission in writing from the publisher.

2–54.
If F
F22 == 150 andu u==55°,
150lblband 55°determine
, determinethe
themagnitude
magnitudeofand the yy
resultant force and clockwise
direction measured its direction measured
from clockwise
the positive x axis offrom
the
the positive
resultant x axis.
force of the three forces acting on the bracket. F3F5 52 lb
3  52 lb

13 13 1212
5 5
SOLUTION
Scalar Notation:
Notation: Suming the force components algebraically, we have
F1F5 80 lb
1  80 lb
xx
:
5
+ F = ©F ;
Rx x FRx = 80 + 52 a b + 150 cos 80°
13 258
25

= 126.05 lb : uu
uu
12 F2F2
+ c FRy = ©Fy; FRy = 52 a b - 150 sin 80°
13

= - 99.72 lb = 99.72 lb T

The magnitude of the resultant force FR is

FR = 2F 2Rx + F 2Ry = 2126.052 + 99.72 2 = 161 lb Ans.

The directional angle u measured clockwise from positive x axis is


FRy 99.72
u = tan - 1 = tan - 1 a b = 38.3° Ans.
FRx 126.05

Ans:
FR = 161 lb
u = 38.3°

75
© 2022 by R. C. Hibbeler. Published by Pearson Education, Inc., Hoboken, NJ. All rights reserved. This material is protected under all copyright laws
as they currently exist. No portion of this material may be reproduced, in any form or by any means, without permission in writing from the publisher.

2–55.

Determine the resultant force acting on the hook, and its y


direction measured clockwise from the positive x axis.

x
SOLUTION
Scalar Notation: Summing the force components along x and y axes algebraically by 5
3
referring to Fig. a, 4 608 F1 5 500 N
+ (F ) = ΣF ;  (F ) = 500 sin 60° - 800 a 4 b
S F2 5 800 N
R x x R x
5
(FR)x = - 206.99 N = 206.99 N d
3
+ c (FR)y = ΣFy;  (FR)y = - 500 cos 60° - 800 a b
5
(FR)y = - 730 N = 730 N T

By referring to Fig. b, the magnitude of the resultant force FR is

FR = 2(FR)2x + (FR)2y = 2206.992 + 7302 = 758.78 N = 759 N Ans.

And the directional angle u of FR measured clockwise from the positive x axis is
(FR)y 730
u a = tan - 1 c d = tan - 1 a b = 74.17°
(FR)x 206.99
Then

u = 180° - a = 180° - 74.17° = 105.83° = 106° Ans.

Ans:
FR = 759 N
u = 106°

76
© 2022 by R. C. Hibbeler. Published by Pearson Education, Inc., Hoboken, NJ. All rights reserved. This material is protected under all copyright laws
as they currently exist. No portion of this material may be reproduced, in any form or by any means, without permission in writing from the publisher.

*2–56.

If the magnitude of the resultant force acting on the bracket y


is to be 450 N directed along the positive u axis, determine F1 u
the magnitude of F1 and its direction f.
f
30
x
SOLUTION F2 200 N

Rectangular Components: By referring to Fig. a, the x and y components of F1, F2,


Rectangular 12 13
F3, and FR can be written as 5

(F1)x = F1 sin f (F1)y = F1 cos f F3 260 N

(F2)x = 200 N ( F2)y = 0

5 12
(F3)x = 260 ¢ ≤ = 100 N (F3)y = 260 ¢ ≤ = 240 N
13 13

(FR)x = 450 cos 30° = 389.71 N (FR)y = 450 sin 30° = 225 N

Resultant Force: Summing the force components algebraically along the x and y axes,

:
+ ©(F ) = ©F ;
R x x 389.71 = F1 sin f + 200 + 100

F1 sin f = 89.71 (1)

+ c ©(FR)y = ©Fy; 225 = F1 cos f - 240

F1 cos f = 465 (2)

Solving Eqs. (1) and (2), yields

f = 10.9° F1 = 474 N Ans.

Ans:
f = 10.9°
F1 = 474 N

77
© 2022 by R. C. Hibbeler. Published by Pearson Education, Inc., Hoboken, NJ. All rights reserved. This material is protected under all copyright laws
as they currently exist. No portion of this material may be reproduced, in any form or by any means, without permission in writing from the publisher.

2–57.

If the resultant force acting on the bracket is required to be y


a minimum, determine the magnitudes of F1 and the F1 u
resultant force. Set f = 30°.
f
30
x
SOLUTION F2 200 N

Rectangular Components: By referring to Fig. a, the x and y components of F1, F2,


Rectangular 12 13
and F3 can be written as 5

(F1)x = F1 sin 30° = 0.5F1 (F1)y = F1 cos 30° = 0.8660F1 F3 260 N

(F2)x = 200 N (F2)y = 0


5 12
(F3)x = 260a b = 100 N (F3)y = 260a b = 240 N
13 13

Resultant Force:
Force: Summing the force components algebraically along the x and y axes,

:
+ ©(F ) = ©F ;
R x x (FR)x = 0.5F1 + 200 + 100 = 0.5F1 + 300

+ c ©(FR)y = ©Fy; (FR)y = 0.8660F1 - 240

The magnitude of the resultant force FR is

FR = 2(FR)x2 + (FR)y2

= 2(0.5F1 + 300)2 + (0.8660F1 - 240)2

= 2F 21 - 115.69F1 + 147 600 (1)

Thus,

FR2 = F 21 - 115.69F1 + 147 600 (2)

The first derivative of Eq. (2) is

dFR
2FR = 2F1 - 115.69 (3)
dF1
dFR
For FR to be minimum, = 0. Thus, from Eq. (3)
dF1

dFR
2FR = 2F1 - 115.69 = 0
dF1
F1 = 57.846 N = 57.8 N Ans.

from Eq. (1),

FR = 2(57.846)2 - 115.69(57.846) + 147 600 = 380 N Ans.

Ans:
FR = 380 N
F1 = 57.8 N

78
© 2022 by R. C. Hibbeler. Published by Pearson Education, Inc., Hoboken, NJ. All rights reserved. This material is protected under all copyright laws
as they currently exist. No portion of this material may be reproduced, in any form or by any means, without permission in writing from the publisher.

2–58. y

Three forces act on the bracket. Determine the magnitude 4 kN F


and direction u of F so that the resultant force is directed
along the positive x′ axis and has a magnitude of 8 kN. 15
u
x'

30
x
6 kN

SOLUTION
Scalar Notation: Equating the force components along the x and y axes algebraically
by referring to Fig. a,
+ (F ) = ΣF ;  8 cos 30° = F sin u + 6 - 4 sin 15°
S R x x

F sin u = 1.9635(1)

+ c (FR)y = ΣFy ;  8 sin 30° = F cos u + 4 cos 15°

F cos u = 0.1363(2)
Divide Eq (1) by (2)
tan u = 14.406 u = 86.03° = 86.0° Ans.
Substitute this result into Eq (1)
F sin 86.03° = 1.9635
F = 1.968 kN = 1.97 kN Ans.

Ans:
 u = 86.0°
F = 1.97 kN

79
© 2022 by R. C. Hibbeler. Published by Pearson Education, Inc., Hoboken, NJ. All rights reserved. This material is protected under all copyright laws
as they currently exist. No portion of this material may be reproduced, in any form or by any means, without permission in writing from the publisher.

2–59.

If F = 5 kN and u = 30°, determine the magnitude of the y


r­esultant force and its direction, measured counter-­
4 kN F
clockwise from the positive x axis.
15
u
x'

30
x
SOLUTION 6 kN

Scalar Notation: Summing the force components along x and y axes algebraically
by referring to Fig. a,
S+ (F ) = ΣF ; (FR)x = 5 sin 30° + 6 - 4 sin 15° = 7.465 kN S
R x x

+ c (FR)y = ΣFy; (FR)y = 4 cos 15° + 5 cos 30° = 8.194 kN c

By referring to Fig. b, the magnitude of the resultant force is

FR = 2(FR)2x + (FR)2y = 27.4652 + 8.1942 = 11.08 kN = 11.1 kN Ans.


And its directional angle u measured counterclockwise from the positive
x axis is
(FR)y 8.194
u = tan - 1 c d = tan - 1 a b = 47.67° = 47.7° Ans.
(FR)x 7.465

Ans:
FR = 11.1 kN
u = 47.7°

80
© 2022 by R. C. Hibbeler. Published by Pearson Education, Inc., Hoboken, NJ. All rights reserved. This material is protected under all copyright laws
as they currently exist. No portion of this material may be reproduced, in any form or by any means, without permission in writing from the publisher.

*2–60.
Determine
The force Fthe
hasmagnitudes of the
a magnitude x, y,
of 80 components
lb zand of the
acts within the z
octantF. shown. Determine the magnitudes of the x, y,
force
z components of F. Fz

F  80 lb

b  45
SOLUTION Fy
y
a  60
2 2 2
1 = cos 60° + cos 45° + cos g Fx

Solving for the positive root, g = 60°


x
Fx = 80 cos 60° = 40.0 lb Ans.

Fy = 80 cos 45° = 56.6 lb Ans.

Fz = 80 cos 60° = 40.0 lb Ans.

Ans:
Fx = 40.0 lb
Fy = 56.6 lb
Fz = 40.0 lb

81
© 2022 by R. C. Hibbeler. Published by Pearson Education, Inc., Hoboken, NJ. All rights reserved. This material is protected under all copyright laws
as they currently exist. No portion of this material may be reproduced, in any form or by any means, without permission in writing from the publisher.

2–61.

The bolt is subjected to the force F, which has components z


acting along the x, y, z axes as shown. If the magnitude of F is
80 N, and a = 60° and g = 45°, determine the magnitudes
of its components.
Fz

g
F b
y
Fy a
SOLUTION
Fx
cosb = 21 - cos2 a - cos2g

= 21 - cos2 60° - cos2 45°


x
b = 120°

Fx = |80 cos 60°| = 40 N Ans.

Fy = |80 cos 120°| = 40 N Ans.

Fz = |80 cos 45°| = 56.6 N Ans.

Ans:
Fx = 40 N
Fy = 40 N
Fz = 56.6 N

82
© 2022 by R. C. Hibbeler. Published by Pearson Education, Inc., Hoboken, NJ. All rights reserved. This material is protected under all copyright laws
as they currently exist. No portion of this material may be reproduced, in any form or by any means, without permission in writing from the publisher.

2–62.

The cable at the end of the beam exerts a force of 450 lb on z


the beam. Express F as a Cartesian vector.

y
608
508
SOLUTION
x
cos2 a + cos2 b + cos2 g = 1
F 5 450 lb
cos2 50° + cos2 60° + cos2 g = 1 cos g = {0.5804

From the figure, cos g = -0.5804 g = 125.48°

Force Vector:
F = 450(cos 50°i + cos 60°j + cos 125.48°k)

= {289i + 225j - 261k} lb Ans.

Ans:
F = {289i + 225j - 261k} lb

83
© 2022 by R. C. Hibbeler. Published by Pearson Education, Inc., Hoboken, NJ. All rights reserved. This material is protected under all copyright laws
as they currently exist. No portion of this material may be reproduced, in any form or by any means, without permission in writing from the publisher.

2–63.

The force F acts on the bracket within the octant shown. If z


F = 400 N, b = 60°, and g = 45°, determine the x, y, z
components of F. g
F

SOLUTION b
a
Coordinate Direction Angles: Since b and g are known, the third angle a can be
determined from
x
2 2
cos a + cos b + cos g = 12 y

cos2 g + cos2 60° + cos2 45° = 1

cos a = ; 0.5

Since F is in the octant shown in Fig. a, ux must be greater than 90°. Thus,
a = cos - 1(- 0.5) = 120°.

Rectangular Components: By referring to Fig. a, the x, y, and z components of F can


be written as

Fx = F cos a = 400 cos 120° = - 200N Ans.

Fy = F cos b = 400 cos 60° = 200N Ans.

Fz = F cos g = 400 cos 45° = 283N Ans.

The negative sign indicates that Fx is directed towards the negative x axis.

Ans:
Fx = - 200 N
Fy = 200 N
Fz = 283 N

84
© 2022 by R. C. Hibbeler. Published by Pearson Education, Inc., Hoboken, NJ. All rights reserved. This material is protected under all copyright laws
as they currently exist. No portion of this material may be reproduced, in any form or by any means, without permission in writing from the publisher.

*2–64.

The force F acts on the bracket within the octant shown. If z


the magnitudes of the x and z components of F are
Fx = 300 N and Fz = 600 N, respectively, and b = 60°, g
F
determine the magnitude of F and its y component. Also,
find the coordinate direction angles a and g.

SOLUTION b
a
Rectangular Components: The magnitude of F is given by
x
F = 2Fx 2 + Fy 2 + Fz 2
y
F = 23002 + Fy 2 + 6002

F 2 = Fy 2 + 450 000 (1)

The magnitude of Fy is given by

Fy = F cos 60° = 0.5F (2)

Solving Eqs. (1) and (2) yields

F = 774.60 N = 775 N Ans.

Fy = 387 N Ans.

Coordinate Direction Angles: Since F is contained in the octant so that Fx is


directed towards the negative x axis, the coordinate direction angle ux is given by

- Fx - 300
a = cos - 1 a b = cos - 1 a b = 113° Ans.
F 774.60

The third coordinate direction angle is

- FZ 600
g = cos - 1 a b = cos - 1 a b = 39.2° Ans.
F 774.60

Ans:
F = 775 N
Fy = 387 N
a = 113°
g = 39.2°

85
© 2022 by R. C. Hibbeler. Published by Pearson Education, Inc., Hoboken, NJ. All rights reserved. This material is protected under all copyright laws
as they currently exist. No portion of this material may be reproduced, in any form or by any means, without permission in writing from the publisher.

2–65.

Express
The screweach
eye isforce in Cartesian
subjected to the twovector
forces form
shown.and then
Express z
F1 = 300 N
determine the
each force in resultant
Cartesian force.
vector Find
form andthe
thenmagnitude
determineand
the
coordinate direction
resultant force. angles
Find the of the resultant
magnitude force. direction
and coordinate
angles of the resultant force.

60°
120°
45°
y
SOLUTION
45°
F1 = 300( - cos 60° sin 45° i + cos 60° cos 45° j + sin 60°k)
60°
= {-106.07 i + 106.07 j + 259.81 k} N x

= {- 106 i + 106 j + 260 k} N Ans.


F2 = 500 N
F2 = 500(cos 60° i + cos 45° j + cos 120° k)

= {250.0 i + 353.55 j - 250.0k} N

= {250 i + 354 j - 250 k} N Ans.

FR = F1 + F2

= - 106.07 i + 106.07 j + 259.81 k + 250.0 i + 353.55 j - 250.0 k

= 143.93 i + 459.62 j + 9.81k

= {144 i + 460 j + 9.81 k} N Ans.

FR = 2143.932 + 459.622 + 9.812 = 481.73 N = 482 N Ans.

FR 143.93i + 459.62j + 9.81k


uFR = = = 0.2988i + 0.9541j + 0.02036k
FR 481.73

cos a = 0.2988 a = 72.6° Ans.

cos b = 0.9541 b = 17.4° Ans.

cos g 0.02036 g 88.8° Ans.

Ans:
F1 = { - 106i + 106j + 260k} N
F2 = {250i + 354j - 250k} N
FR = {144i + 460j + 9.81k} N
FR = 482 N
a = 72.6°
b = 17.4°
g = 88.8°

86
© 2022 by R. C. Hibbeler. Published by Pearson Education, Inc., Hoboken, NJ. All rights reserved. This material is protected under all copyright laws
as they currently exist. No portion of this material may be reproduced, in any form or by any means, without permission in writing from the publisher.

2–66.

Determine the coordinate direction angles of F1. z


F1 = 300 N

60°
120°
45°
y
SOLUTION
45°
F1 = 300(- cos 60° sin 45° i + cos 60° cos 45° j + sin 60° k)
60°
= { - 106.07 i + 106.07 j + 259.81 k} N x

= { -106 i + 106 j + 260 k} N


F2 = 500 N
F1
u1 = = - 0.3536 i + 0.3536 j + 0.8660 k
300

a1 = cos-1 (- 0.3536) = 111° Ans.

b 1 = cos-1 (0.3536) = 69.3° Ans.

g1 = cos-1 (0.8660) = 30.0° Ans.

Ans:
a1 = 111°
b1 = 69.3°
g1 = 30.0°

87
© 2022 by R. C. Hibbeler. Published by Pearson Education, Inc., Hoboken, NJ. All rights reserved. This material is protected under all copyright laws
as they currently exist. No portion of this material may be reproduced, in any form or by any means, without permission in writing from the publisher.

2–67.

The two forces F1 and F2 have a resultant force of z


FR = 5 - 100k6 lb. Determine the magnitude and
coordinate direction angles of F2.
The two forces F1 and F2 acting at A have a resultant force B z
of FR = 5 -100k6 lb. Determine the magnitude and
308
coordinate direction angles of F2.
B
y
A 508
30
SOLUTION y
x 50
Cartesian Vector
Cartesion Vector Notation:
Notation: A F1 5 60 lb
F2
FR = { -100k} lb
x
F1  60 lb
F1 = 60{ -cos 50°cos 30°i + cos 50°sin 30°j - sin 50°k} lb F2

= { -33.40i + 19.28j - 45.96k} lb

F2 = {F2xi + F2y j + F2z k} lb

Resultant Force:
Resultant Force:

FR = F1 + F2

- 100k = {(F2x - 33.40) i + (F2y + 19.28)j + (F2z - 45.96) k}

Equating i, j and k components, we have

F2x - 33.40 = 0 F2x = 33.40 lb

F2y + 19.28 = 0 F2y = - 19.28 lb

F2z - 45.96 = - 100 F2z = - 54.04 lb

The magnitude of force F2 is

F2 = 2F 22x + F 22y + F 22z

= 233.402 + (- 19.28)2 + ( - 54.04)2

= 66.39 lb = 66.4 lb Ans.

The coordinate direction angles for F2 are

F2x 33.40
cos a = = a = 59.8° Ans.
F2 66.39
F2y -19.28
cos b = = b = 107° Ans.
F2 66.39
F2z - 54.04
cos g = = g = 144° Ans.
F2 66.39

Ans:
F2 = 66.4 lb
a = 59.8°
b = 107°
g = 144°

88
© 2022 by R. C. Hibbeler. Published by Pearson Education, Inc., Hoboken, NJ. All rights reserved. This material is protected under all copyright laws
as they currently exist. No portion of this material may be reproduced, in any form or by any means, without permission in writing from the publisher.

*2–68.

Determine thethecoordinate
coordinatedirection
direction angles
angles F1 and
of force
of the F1 z
indicate
and themthem
indicate on the
on figure.
the figure.

30
SOLUTION y
A 50
Unit Vector For
Unit Vector Foce FF1: :
For Force 1

uF1 = - cos 50°cos 30°i + cos 50°sin 30°j - sin 50°k


x
F1  60 lb
= - 0.5567i + 0.3214j - 0.7660k F2

Coordinate Direction
Direction Angles:
Angles: From the unit vector obtained above, we have

cos a = - 0.5567 a = 124° Ans.

cos b = 0.3214 b = 71.3° Ans.

cos g = - 0.7660 g = 140° Ans.

Ans:
a = 124°
b = 71.3°
g = 140°

89
© 2022 by R. C. Hibbeler. Published by Pearson Education, Inc., Hoboken, NJ. All rights reserved. This material is protected under all copyright laws
as they currently exist. No portion of this material may be reproduced, in any form or by any means, without permission in writing from the publisher.

2–69.

The bracket is subjected to the two forces shown. Express z


each force in Cartesian vector form and then determine the
resultant force FR. Find the magnitude and coordinate F2 = 400 N
direction angles of the resultant force. 60°

45°
120°
SOLUTION y
25°
Cartesian Vector
Cartesion Vector Notation:
Notation:

F1 = 2505cos 35° sin 25°i + cos 35° cos 25°j - sin 35°k6 N 35°
x
= 586.55i + 185.60j - 143.39k6 N
F1 = 250 N
= 586.5i + 186j - 143k6 N Ans.

F2 = 4005cos 120°i + cos 45°j + cos 60°k6 N

= 5 -200.0i + 282.84j + 200.0k6 N

= 5-200i + 283j + 200k6 N Ans.

Resultant Force:
Force:

FR = F1 + F2

= 5186.55 - 200.02i + 1185.60 + 282.842j + 1- 143.39 + 200.02 k6

= 5 - 113.45i + 468.44j + 56.61k6 N

= 5- 113i + 468j + 56.6k6 N Ans.

The magnitude of the resultant force is

FR = 2F2Rx + F2Ry + F2Rz

= 21 -113.4522 + 468.442 + 56.612

= 485.30 N = 485 N Ans.

The coordinate direction angles are

FRx -113.45
cos a = = a = 104° Ans.
FR 485.30
F Ry 468.44
cos b = = b = 15.1° Ans.
FR 485.30
FRz 56.61
cos g = = g = 83.3° Ans.
FR 485.30

Ans:
F1 = {86.5i + 186j - 143k} N
F2 = { - 200i + 283j + 200k} N
FR = { - 113i + 468j + 56.6k} N
FR = 485 N
a = 104°
b = 15.1°
g = 83.3°

90
© 2022 by R. C. Hibbeler. Published by Pearson Education, Inc., Hoboken, NJ. All rights reserved. This material is protected under all copyright laws
as they currently exist. No portion of this material may be reproduced, in any form or by any means, without permission in writing from the publisher.

2–70.

Determine the magnitude and coordinate direction angles z


of the resultant force, and sketch this vector on the
coordinate system.
F2  525 N
60
120
45
y
4 5
x
3

F1  450 N

SOLUTION
Cartesian Vector Notation: For F1 and F2,
3 4
F1 = 450 a j - kb = {270j - 360k} N
5 5

F2 = 525 (cos 45°i + cos 120°j + cos 60°k) = {371.23i - 262.5j + 262.5k} N

Resultant Force:
FR = F1 + F2

= {270j - 360k} + {371.23i - 262.5j + 262.5k}

= {371.23i + 7.50j - 97.5k} N

The magnitude of the resultant force is

FR = 2(FR)2x + (FR)2y + (FR)2z = 2371.232 + 7.502 + ( - 97.5)2

= 383.89 N = 384 N Ans.


The coordinate direction angles are
(FR)x 371.23
cos a = = ; a = 14.76° = 14.8° Ans.
FR 383.89

(FR)y 7.50
cos b = = ; b = 88.88° = 88.9° Ans.
FR 383.89

(FR)z - 97.5
cos g = = ; g = 104.71° = 105° Ans.
FR 383.89

Ans:
FR = 384 N
a = 14.8°
b = 88.9°
g = 105°

91
© 2022 by R. C. Hibbeler. Published by Pearson Education, Inc., Hoboken, NJ. All rights reserved. This material is protected under all copyright laws
as they currently exist. No portion of this material may be reproduced, in any form or by any means, without permission in writing from the publisher.

2–71.

Determine
Specify the the magnitude
magnitude andand coordinate
coordinate direction
direction angles
angles a1, z
a , b1, g of
b 1, g1 of F1 so
1 1 F so that the resultant of the three forces
1 that the resultant of the three forces acting acting
on
on the
the bracket
bracket isis FFRR == 55-
- 350k6
350k6 lb. Note that
lb. Note F3 lies
that F
3
lies in
in the
the
x–y plane.
x–y plane. F3 = 400 lb
γ1

30°
y
F2 = 200 lb β1
SOLUTION
α1
F1
F1 = Fx i + Fy j + Fz k

F2 = - 200 j x

F3 = - 400 sin 30° i + 400 cos 30° j

= - 200 i + 346.4 j

FR = ©F

-350 k = Fx i + Fy j + Fz k - 200 j - 200 i + 346.4 j

0 = Fx - 200 ; Fx = 200 lb

0 = Fy - 200 + 346.4 ; Fy = - 146.4 lb

Fz = - 350 lb

F1 = 2(200)2 + (-146.4)2 + (- 350)2

F1 = 425.9 lb = 429 lb Ans.

200
a1 = cos-1 a b = 62.2° Ans.
428.9

- 146.4
b 1 = cos-1 a b = 110° Ans.
428.9

- 350
g1 = cos-1 = 145° Ans.
428.9

Ans:
F1 = 429 lb
a1 = 62.2°
b1 = 110°
g1 = 145°

92
© 2022 by R. C. Hibbeler. Published by Pearson Education, Inc., Hoboken, NJ. All rights reserved. This material is protected under all copyright laws
as they currently exist. No portion of this material may be reproduced, in any form or by any means, without permission in writing from the publisher.

*2–72.

Two forces F1 and F2 act on the screw eye. If the resultant z


force FR has a magnitude of 150 lb and the coordinate
direction angles shown, determine the angle (g) and the F2 FR  150 lb
magnitude of F2 and its coordinate direction angles. g
130
y
F1  80 lb
120

SOLUTION
Cartesian Vector Notation: For FR, g can be determined from
cos2 a + cos2 b + cos2 g = 1

cos2 120° + cos2 50° + cos2 g = 1


cos g = {0.5804

Here g 6 90°, then

g = 54.52° Ans.

Thus
FR = 150(cos 120°i + cos 50°j + cos 54.52°k)

= { - 75.0i + 96.42j + 87.05k} lb

Also
F1 = {80j} lb

Resultant Force:
FR = F1 + F2

{ -75.0i + 96.42j + 87.05k} = {80j} + F2

F2 = { - 75.0i + 16.42j + 87.05k} lb

Thus, the magnitude of F2 is

F2 = 2(F2)x + (F2)y + (F2)z = 2( - 75.0)2 + 16.422 + 87.052

= 116.07 lb = 116 lb Ans.


And its coordinate direction angles are
(F2)x - 75.0
cos a2 = = ; a2 = 130.25° = 130° Ans.
F2 116.07

(F2)y 16.42
cos b2 = = ; b2 = 81.87° = 81.9° Ans.
F2 116.07

(F2)z 87.05
cos g2 = = ; g2 = 41.41° = 41.4° Ans.
F2 116.07

Ans:
    g = 54.52°
    FR = 116 lb
 cos a2 = 130°
cos b 2 = 81.9°
 cos g2 = 41.4°

93
© 2022 by R. C. Hibbeler. Published by Pearson Education, Inc., Hoboken, NJ. All rights reserved. This material is protected under all copyright laws
as they currently exist. No portion of this material may be reproduced, in any form or by any means, without permission in writing from the publisher.

2–73. z

Express each force in Cartesian vector form.


F3  200 N
F2  150 N

F1  90 N
5
60
y
3 4
45

SOLUTION
x
Cartesian Vector Notation: For F1, F2 and F3,
4 3
F1 = 90 a i + kb = {72.0i + 54.0k} N Ans.
5 5
F2 = 150 (cos 60° sin 45°i + cos 60° cos 45°j + sin 60°k)

= {53.03i + 53.03j + 129.90k} N

= {53.0i + 53.0j + 130k} N Ans.

F3 = {200 k} Ans.

Ans:
F1 = {72.0i + 54.0k} N
F2 = {53.0i + 53.0j + 130k} N
F3 = {200 k}

94
© 2022 by R. C. Hibbeler. Published by Pearson Education, Inc., Hoboken, NJ. All rights reserved. This material is protected under all copyright laws
as they currently exist. No portion of this material may be reproduced, in any form or by any means, without permission in writing from the publisher.

2–74. z

Determine the magnitude and coordinate direction angles


of the resultant force, and sketch this vector on the F3  200 N
coordinate system. F2  150 N

F1  90 N
5
60
y
3 4
45

SOLUTION
x
Cartesian Vector Notation: For F1, F2 and F3,
4 3
F1 = 90 a i + kb = {72.0i + 54.0k} N
5 5

F2 = 150 (cos 60° sin 45°i + cos 60° cos 45°j + sin 60°k)

= {53.03i + 53.03j + 129.90k} N

F3 = {200 k} N

Resultant Force:
F = F1 + F2 + F3

= (72.0i + 54.0k) + (53.03i + 53.03j + 129.90k) + (200k)

= {125.03i + 53.03j + 383.90} N

The magnitude of the resultant force is

FR = 2(FR)2x + (FR)2y + (FR)2z = 2125.032 + 53.032 + 383.902

= 407.22 N = 407 N  Ans.


And the coordinate direction angles are
(FR)x 125.03
cos a = = ; a = 72.12° = 72.1° Ans.
FR 407.22

(FR)y 53.03
cos b = = ; b = 82.52° = 82.5° Ans.
FR 407.22

(FR)z 383.90
cos g = = ; g = 19.48° = 19.5° Ans.
FR 407.22

Ans:
FR = 407 N
 a = 72.1°
 b = 82.5°
 g = 19.5°

95
© 2022 by R. C. Hibbeler. Published by Pearson Education, Inc., Hoboken, NJ. All rights reserved. This material is protected under all copyright laws
as they currently exist. No portion of this material may be reproduced, in any form or by any means, without permission in writing from the publisher.

2–75.
The
The spur
spur gear
gear isissubjected
subjectedtotothe
thetwo forces.
two Express
forces causedeach
by z
force as a Cartesian vector.
contact with other gears. Express each force as a Cartesian
vector. 60
F2 180 lb

SOLUTION 60 135

7 24 y
F1 = (50)j - (50)k = {14.0j - 48.0k} lb Ans.
25 25 x
25

F2 = 180 cos 60°i + 180 cos 135°j + 180 cos 60°k 24


7
= {90i - 127j + 90k} lb Ans. F1 50 lb

Ans:
F1 = {14.0j - 48.0k} lb
F2 = {90i - 127j + 90k} lb

96
© 2022 by R. C. Hibbeler. Published by Pearson Education, Inc., Hoboken, NJ. All rights reserved. This material is protected under all copyright laws
as they currently exist. No portion of this material may be reproduced, in any form or by any means, without permission in writing from the publisher.

*2–76.
Determine
The spur gearthe resultant of the
is subjected to two forces
the two and express
forces caused the
by z
result as a Cartesian vector.
contact with other gears. Determine the resultant of the two
forces and express the result as a Cartesian vector. 60
F2 180 lb

SOLUTION 60 135

FRx = 180 cos 60° = 90 y


x
7 25
FRy = (50) + 180 cos 135° = -113
25 24
7
24 F1 50 lb
FRz = - (50) + 180 cos 60° = 42
25

FR = {90i - 113j + 42k} lb Ans.

Ans:
FR = {90i - 113j + 42k} lb

97
© 2022 by R. C. Hibbeler. Published by Pearson Education, Inc., Hoboken, NJ. All rights reserved. This material is protected under all copyright laws
as they currently exist. No portion of this material may be reproduced, in any form or by any means, without permission in writing from the publisher.

2–77.

Specify the magnitude F3 and directions a3, b3, and g3 of F3 so z


that the resultant force of the three forces is FR = {9 j} kN.
F2 10 kN
F3
13 5
g3
b3 12

a3
y
30

SOLUTION F1 12 kN
F1 = 12 cos 30° j - 12 sin 30° k = 10.392 j - 6 k x

12 5
F2 = - (10) i + (10) k = -9.231 i + 3.846 k
13 13

Require

FR = F1 + F2 + F3

9 j = 10.392 j - 6 k - 9.231 i + 3.846 k + F3

F3 = 9.231 i - 1.392 j + 2.154 k

Hence,

F3 = 2(9.231)2 + ( - 1.392)2 + (2.154)2

F3 = 9.581 kN = 9.58 kN Ans.

9.231
a3 = cos-1 a b = 15.5° Ans.
9.581

-1.392
b3 = cos-1 a b = 98.4° Ans.
9.581

2.154
g3 = cos-1 a b = 77.0° Ans.
9.581

Ans:
F3 = 9.58 kN
a3 = 15.5°
b3 = 98.4°
g3 = 77.0°

98
© 2022 by R. C. Hibbeler. Published by Pearson Education, Inc., Hoboken, NJ. All rights reserved. This material is protected under all copyright laws
as they currently exist. No portion of this material may be reproduced, in any form or by any means, without permission in writing from the publisher.

2–78.

Determine the coordinate angle g for F2 and then express z


F1  450 N
each force acting on the bracket as a Cartesian vector.

45
SOLUTION 30

Rectangular Components: Since cos2 a2 + cos2 b 2 + cos2g2 = 1, then 45


60 y
cos g2z = ; 21 - cos2 45° - cos2 60° = ; 0.5. However, it is required that
x
g2 7 90°, thus, g2 = cos - 1( -0.5) = 120°. By resolving F1 and F2 into their x, y, and
z components, as shown in Figs, a and b, respectively F1 and F2 can be expressed in
F2  600 N
Cartesian vector form as

F1 = 450 cos 45° sin 30°(- i) + 450 cos 45° cos 30°(+j) + 450 sin 45°(+ k)

= { -159i + 276j + 318k}N Ans.

F2 = 600 cos 45°i + 600 cos 60°j + 600 cos 120°k

= {424i + 300j - 30k}N Ans.

Ans:
F1 = { - 159i + 276j + 318k}N
F2 = {424i + 300j + 30k}N

99
© 2022 by R. C. Hibbeler. Published by Pearson Education, Inc., Hoboken, NJ. All rights reserved. This material is protected under all copyright laws
as they currently exist. No portion of this material may be reproduced, in any form or by any means, without permission in writing from the publisher.

2–79.

Determine the magnitude and coordinate direction angles z


F1  450 N
of the resultant force acting on the bracket.

45
SOLUTION 30

Rectangular Components: Since cos2 a2 + cos2 b 2 + cos2g2 = 1, then 45


60 y
cos g2z = ; 21 - cos2 45° - cos2 60° = ; 0.5. However, it is required that
x
a2 7 90°, thus, g2 = cos - 1( -0.5) = 120°. By resolving F1 and F2 into their x, y, and
z components, as shown in Figs. a and b, respectively, F1 and F2, can be expressed in
F2  600 N
Cartesian vector form, as

F1 = 450 cos 45° sin 30°(-i) + 450 cos 45° cos 30°(+ j) + 450 sin 45°(+ k)

= { -159.10i + 275.57j + 318.20k}N Ans.

F2 = 600 cos 45°i + 600 cos 60°j + 600 cos 120°k

= {424i + 300j - 300k} N Ans.

Resultant Force: By adding F1 and F2 vectorally, we obtain FR.

FR = F1 + F2

= (-159.10i + 275.57j + 318.20k) + (424.26i + 300j - 300k)

= {265.16i + 575.57j + 18.20k} N

The magnitude of FR is

FR = 2(FR)x 2 + (FR)y 2 + (FR)z 2

= 2265.162 + 575.572 + 18.202 = 633.97 N = 634N Ans.

The coordinate direction angles of FR are

a = cos - 1 B R = cos - 1 ¢ ≤ = 65.3°


(FR)x 265.16
Ans.
FR 633.97
(FR)y
b = cos - 1 B R = cos - 1 ¢ ≤ = 24.8°
575.57
Ans.
FR 633.97

(FR)z
g = cos - 1 B R = cos - 1 ¢ ≤ = 88.4°
18.20
Ans.
FR 633.97

Ans:
FR = 634 N
a = 65.3°
b = 24.8°
g = 88.4°

100
© 2022 by R. C. Hibbeler. Published by Pearson Education, Inc., Hoboken, NJ. All rights reserved. This material is protected under all copyright laws
as they currently exist. No portion of this material may be reproduced, in any form or by any means, without permission in writing from the publisher.

*2–80.

Three forces act on the ring. Determine the magnitude and z


coordinate direction angles of the resultant force.
F3 5 100 N

608 608 F1 5 400 N


F2 5 500 N

458
SOLUTION y
Force Vector: 1208

F1 = 400 (cos 120°i + cos 45°j + cos 60°k)


308
= { - 200i + 282.84j + 200k} N

F2 = 500 (sin 60°cos 30° i - sin 60° sin 30°j + cos 60°k) x
= {375i - 216.51j + 250k} N

F3 = {100k} N

Resultant Force Vector:


FR = F1 + F2 + F3

= { - 200i + 282.84j + 200k} + {375i - 216.51j + 250k} + {100k}

= {175i + 66.33j + 550k} N

Magnitude of FR :

FR = 21752 + 66.332 + 5502 = 580.97 N = 581 N Ans.


Coordinate Direction Angles:

FR 175i + 66.33j + 550k


uFR = =
FR 580.97
= 0.3012i + 0.1142j + 0.9467k

cos a = 0.3012 a = 72.5° Ans.

cos b = 0.1142 b = 83.4° Ans.

cos g = 0.9467 g = 18.8° Ans.

Ans:
FR = 581 N
a = 72.5°
b = 83.4°
g = 18.8°

101
© 2022 by R. C. Hibbeler. Published by Pearson Education, Inc., Hoboken, NJ. All rights reserved. This material is protected under all copyright laws
as they currently exist. No portion of this material may be reproduced, in any form or by any means, without permission in writing from the publisher.

2–81.

If the coordinate direction angles for F3 are a3 = 120°, z


b 3 = 60° and g3 = 45°, determine the magnitude and
coordinate direction angles of the resultant force acting on F3  800 lb
the eyebolt.

5
F2  600 lb
SOLUTION 4 3

Force Vectors:
Vector: By resolving F1, F2 and F3 into their x, y, and z components, as shown 30
in Figs. a, b, and c, respectively, F1, F2 and F3 can be expressed in Cartesian vector y
x
form as
F1  700 lb
F1 = 700 cos 30°(+ i) + 700 sin 30°(+j) = 5606.22i + 350j6 lb

4 3
F2 = 0i + 600 a b ( +j) + 600 a b ( + k) = 5480j + 360k6 lb
5 5

F3 = 800 cos 120°i + 800 cos 60°j + 800 cos 45°k = 3-400i + 400j + 565.69k4 lb

Force: By adding F1, F2 and F3 vectorally, we obtain FR. Thus,


Resultant Force:
Resultant

FR = F1 + F2 + F3

= (606.22i + 350j) + (480j + 360k) + ( - 400i + 400j + 565.69k)

= 3206.22i + 1230j + 925.69k4 lb

The magnitude of FR is

FR = 3(FR)x 2 + (FR)y 2 + (FR)z 2

= 3(206.22)2 + (1230)2 + (925.69)2 = 1553.16 lb = 1.55 kip Ans.

The coordinate direction angles of FR are


(FR)x 206.22
a = cos-1 c d = cos-1 a b = 82.4° Ans.
FR 1553.16
(FR)y 1230
b = cos-1 c d = cos-1 a b = 37.6° Ans.
FR 1553.16

(FR)z 925.69
g = cos-1 c d = cos-1 a b = 53.4° Ans.
FR 1553.16

Ans:
FR = 1.55 kip
 a = 82.4°
    b = 37.6°
    g = 53.4°

102
© 2022 by R. C. Hibbeler. Published by Pearson Education, Inc., Hoboken, NJ. All rights reserved. This material is protected under all copyright laws
as they currently exist. No portion of this material may be reproduced, in any form or by any means, without permission in writing from the publisher.

2–82.

If the coordinate direction angles for F3 are a3 = 120°, z


b 3 = 45° and g3 = 60°, determine the magnitude and
coordinate direction angles of the resultant force acting on F3  800 lb
the eyebolt.

5
4 3 F2  600 lb
SOLUTION
Force Vectors: By resolving F1, F2 and F3 into their x, y, and z components, as shown 30
in Figs. a, b, and c, respectively, F1, F2, and F3 can be expressed in Cartesian vector x y
form as F1  700 lb

F1 = 700 cos 30°( + i) + 700 sin 30°(+j) = 5606.22i + 350j6 lb

4 3
F2 = 0i + 600 a b (+ j) + 600 a b( + k) = 5480j + 360k6 lb
5 5

F3 = 800 cos 120°i + 800 cos 45°j + 800 cos 60°k = 5- 400i + 565.69j + 400k6 lb

FR = F1 + F2 + F3

= 606.22i + 350j + 480j + 360k - 400i + 565.69j + 400k

= 5206.22i + 1395.69j + 760k6 lb

FR = 3(206.22)2 + (1395.69)2 + (760)2


= 1602.52 lb = 1.60 kip Ans.

206.22
a = cos-1 a b = 82.6° Ans.
1602.52

1395.69
b = cos-1 a b = 29.4° Ans.
1602.52

760
g = cos-1 a b = 61.7° Ans.
1602.52

Ans:
FR = 1.60 kip
   a = 82.6°
   b = 29.4°
   g = 61.7°

103
© 2022 by R. C. Hibbeler. Published by Pearson Education, Inc., Hoboken, NJ. All rights reserved. This material is protected under all copyright laws
as they currently exist. No portion of this material may be reproduced, in any form or by any means, without permission in writing from the publisher.

If the direction of the resultant force acting on the eyebolt


2–83. z
is defined by the unit vector uFR = cos 30°j + sin 30°k, zz
If
If the
the direction
determine of
of the
the resultant
the coordinate force
direction acting
acting on
force angles the
of F eyebolt
direction resultant on 3 and
the the
eyebolt F3  800 lb
is defined
magnitude
is defined ofby the unit vector u = +
. unit vector uFRR = cos 30°j + sin 30°k
byFRthe F cos 30°j sin 30°k,,
determine
determine the
the coordinate
coordinate direction
direction angles
angles of F3 and
of F the F3  800 lb
lb
3 and the F  800
3
magnitude
magnitude ofof F
FRR..

5
4 3 F2  600 lb
SOLUTION 5
5
30 4 3 F  600
F22  600 lb
lb
SOLUTION
Force Vectors: By resolving F1, F2 and F3 into their x, y, and z components, as shown
SOLUTION
4 3

in Figs. a, b, and c, respectively, F1, F2, and F3 can be expressed in Cartesian vector x 30 y
Force Vectors: F1,, F
F2 and F3 into their x, y, and z components, as shown 30
Forceas
form Vectors: By
By resolving
resolving F 1 2 and F3 into their x, y, and z components, as shown F1  700 lb
in a, b, c, F , F
in Figs. a, b, and c, respectively, F1, F2, and
Figs. and respectively, , and FF3 can be expressed in Cartesian vector yy
3 can be expressed in Cartesian vector xx
1 2
form
F
form
1 = as
700
as cos 30°( + i) + 700 sin 30°(+j) = 5606.22i + 350j6 lb F 700 lb
 700 lb
F11 
F1 =
F = 700
700 cos
cos 30°(
30°(
4+ i) +
+ i) + 700
700 sin
sin 330°(
30°(+j)
+j) = 5606.22i
= 5606.22i + 350j6
+ 350j6 lb
lb
F 2 = 0i + 600a b( +j) + 600 a b ( +k) = 5480j +
1 360k6 lb
5 5
4 3
F2 =
F = 0i 600a 4 b(
+ 600a
0i + b( +j)
+j) ++ 600 aa 3 b(
b (+ k) =
+k) 5480j +
= 5480j + 360k6 lb
lb
F 32 = 800 cos a3i5
5+ 800 cos600
b3 j +55 800 cos g3k
360k6

F3 == 800
800 cos aa3ii +
+ of
800 cos
FRcos b 3 jj +
+ 800
800 cos
by ucos g3k
FR =
k
Since
F 3 the direction
cos 3 800 is defined
b 3 g 3 cos 30°j + sin 30°k, it can be written in
Cartesian vector form as
Since
Since the
the direction
direction of FR is
of F defined by uF = = cos
cos 30°j + sin
30°j + sin 30°k
30°k,, it
it can
can be
be written
written in
R is defined by uFR in
F R = FRuFvector
Cartesian
Cartesian = FRform
vector
R form as + sin 30°k) = 0.8660FR j + 0.5FR k
(cos 30°j
as R

FR == F uFR =
FRu
Resultant = F (cos
FRBy +F sin
30°j + sin 30°k) 0.8660FR j +
= vectorally, + 0.5F
0.5FRk
F R R FForce:
R
adding
R(cos 30°j 1, F230°k)
, and F=
3 0.8660FR j we R k FR. Thus,
obtain

Resultant
F Force:
R = F1 +Force:
Resultant F2 + FBy adding F , F , and F vectorally, we obtain FR.. Thus,
3 adding F11, F22, and F33 vectorally, we obtain FR
By Thus,

F0.8660F
F F1R+
= F +j +F + FR3 k = (606.22i + 350j) + (480j + 360k) + (800 cos a3i + 800 cos b 3 j + 800 cos g3k)
F20.5F
2 + F3
R =
R 1
0.8660FRR j + 0.5FRR k = (606.22
(606.22i ++
+ 800
350j) )i + (350
cos+a3(480j + 480
+ 360k)
360k) + 800
+ (800
(800 cos
cos a3bii 3+ + (360
+)j 800
800 b+3 jj800
cos b cos gcos
+ 800 3)kg3k)
0.8660FR j + 0.5FR k = (606.22i 350j) + (480j + + cos a 3 cos 3 + 800 cos g3k)
0.8660F
Equating jj +
0.8660FRthe
R
+ i,0.5F
0.5F
j, and
Rkkk=
R
=components,
(606.22 +
(606.22 + 800
800 cos a
we have
cos a3)i
3
+ (350
)i + + 480
(350 + + 800
480 + 800 cos
cos bb 3)j
3)j + (360 +
+ (360 + 800800 cos
cos gg3)k
3)k
0Equating
= 606.22
Equating the+ ii,,800
the jj,, and
cosk
and kacomponents,
components,
3 we
we have
have
800 cos a3 = -606.22 (1)
= 606.22
00 = 606.22 + + 800800 cos cos a
a3
0.8660F
800 cos R 3 == 350
a -606.22 + 480 3+ 800 cos b 3 (1)
800 cos a = -606.22 (1)
800 cos b33 = 0.8660FR - 830 (2)
0.8660F
0.8660FR R = = 350
350 + + 480
480 + + 800
800 cos
cos b
b3
3
0.5F
800 R =b360
cos = + 800 cos
0.8660F -
g3830 (2)
800 cos b 3 = 0.8660FR - 830
3 R (2)
800 cos g3 = 0.5FR - 360 (3)
0.5FR =
0.5F = 360
360 + + 800 800 cos
cos gg3
R 3
800 cos
Squaring g = 0.5F - 360
800 cos g3 = 0.5FR - 360Eqs. (1), (2), and (3), yields
3and then Radding (3)
(3)
Squaring
8002
[cos2and
Squaring a3 +then
and cosadding
then
2
b 3 + Eqs.
adding cos2 (1),
Eqs. g3] (2),
(1), = Fand
(2),
2 (3), yields
R -(3),
and yields R + 1,186,000
1797.60F (4)
2 2 2
However,
[cos22 cos
80022 [cos 3 +2 cos 3 +2 cos F=R 221-
. Thus, fromREq.
+ (4)
2 2
a3 ++acos
cos b3 + +b cos
cos g3]] ==g3F - 1797.60F 1,186,000 (4)
(4)
800 a 3 b 3 g3 R 1797.60F R + 1,186,000
2
R - 1797.60F 2 b 3 += cos
546,000 0 g3 = 1. Thus, from Eq. (4)
F 2 2 2
However, cos2 a + cos
aR3 + cos b 3 + cos2 g
However, cos 3 3 = 1. Thus, from Eq. (4)
2
FR 2 -- 1797.60F
Solving
F 1797.60F
the above + 546,000
R quadratic
+ = 00
546,000equation,
= we have two positive roots
R R

Solving
F the
the above
R = 387.09
Solving quadratic
N = 387
above N
quadratic equation,
equation, we
we have
have two
two positive
positive roots
roots Ans.
FRR = 1410.51
387.09 N ==387
NN= 1.41NkN
387 Ans.
FR = 387.09 N Ans.
FR =
From
F = Eq.
1410.51
(1), N
1410.51 = 1.41
N = 1.41 kN
kN Ans.
Ans.
R

From
a Eq.
Eq. (1),
3 = 139°
From (1), Ans.

= 139°
139° FR = 387.09 N into Eqs. (2), and (3), yields
Substituting
a3 =
a Ans.
Ans.
3

3 = 128° FR
Substituting = 387.09 N into g3 = (2),
102° (3), yields Ans.
387.09 N into Eqs.
Eqs. (2), and Ans:
b
Substituting FR = and (3), yields
Substituting F = 1410.51 N intog3Eqs. (2), and (3), yields a3 = 139°
b = 128°
b 33 = 128° R = 102°
g3 = 102° Ans.
Ans. b3 = 128°, g3 = 102°, if FR = 387 N
3 = 60.7° FR
Substituting = = 64.4°and (3), yields Ans. b3 = 60.7°, g3 = 64.4°, if FR = 1.41 kN
= 1410.51 N into Eqs. (2),
into
b 1410.51 N g3Eqs.
Substituting F R (2), and (3), yields
b3 =
b = 60.7°
60.7° g = 64.4°
g33 = 64.4° Ans.
Ans.
3 104
© 2022 by R. C. Hibbeler. Published by Pearson Education, Inc., Hoboken, NJ. All rights reserved. This material is protected under all copyright laws
as they currently exist. No portion of this material may be reproduced, in any form or by any means, without permission in writing from the publisher.

*2–84.

The pipe is subjected to the force F, which has components z


acting along the x, y, z axes. If the magnitude of F is 12 kN,
and a = 120° and g = 45°, determine the magnitudes of its
Fz
three components.

F
Fx

x
Fy
SOLUTION
y
cos2 a + cos2 b + cos2 g = 1

cos2 120° + cos2 b + cos2 45° = 1 cos b = {0.5

From the figure, cos b = + 0.5  b = 60°


Fx = - F cos a = |12 cos 120°| = 6 kN Ans.
Fy = - F cos b = |12 cos 60°| = 6 kN Ans.
Fz = - F cos g = |12 cos 45°| = 8.49 kN Ans.

Ans:
Fx = 6 kN
Fy = 6 kN
Fz = 8.49 kN

105
© 2022 by R. C. Hibbeler. Published by Pearson Education, Inc., Hoboken, NJ. All rights reserved. This material is protected under all copyright laws
as they currently exist. No portion of this material may be reproduced, in any form or by any means, without permission in writing from the publisher.

2–85.

The pipe is subjected to the force F, which has components


Fx = 1.5 kN and Fz = 1.25 kN. If b = 75°, determine the
magnitude of F and Fy. z

Fz

F
Fx

SOLUTION
Fx = F cos a = 1.5[1] x
Fy
Fz = F cos g = 1.25[2] y

Squaring Eqs.[1] and [2] and rearranging yields:

2.25
cos2 a = [3]
F2
1.5625
cos2 g = [4]
F2

cos2 a + cos2 b + cos2 g = 1

2.25 1.5625
+ cos275° + = 1
F2 F2

F = 2.021 kN = 2.02 kN Ans.


Fy = F cos b = 2.021 cos 75° = 0.523 kN Ans.

Ans:
F = 2.02 kN
Fy = 0.523 kN

106
© 2022 by R. C. Hibbeler. Published by Pearson Education, Inc., Hoboken, NJ. All rights reserved. This material is protected under all copyright laws
as they currently exist. No portion of this material may be reproduced, in any form or by any means, without permission in writing from the publisher.

2–86.

Determine the length of the connecting rod AB by first y


formulating a position vector from A to B and then
determining its magnitude.

300 mm

SOLUTION
Position Vector: The coordinates of points A and B are A( -150 cos 30°, O x
- 150 sin 30°) mm and B(0, 300) mm respectively. Then
30 A
rAB = [0 - ( - 150 cos 30°)]i + [300 - ( - 150 sin 30°)]j

= {129.90i + 375j} mm 150 mm

Thus, the magnitude of rAB is

rAB = 2129.902 + 3752 = 396.86 mm = 397 mm Ans.

Ans:
rAB = 397 mm

107
© 2022 by R. C. Hibbeler. Published by Pearson Education, Inc., Hoboken, NJ. All rights reserved. This material is protected under all copyright laws
as they currently exist. No portion of this material may be reproduced, in any form or by any means, without permission in writing from the publisher.

2–87.

Determine the distance between the end points A and B on z


the pipe assembly.

A 458

2 ft
y
SOLUTION
4 ft
Position Vector:
r = (4 - 0)i + (3 - 0)j + ( -2 - 2)k 3 ft 2 ft

= {4i + 3j - 4k} ft B
x
Magnitude:

r = 242 + 32 + ( - 4)2 = 6.40 ft Ans.

Ans:
r = 6.40 ft

108
© 2022 by R. C. Hibbeler. Published by Pearson Education, Inc., Hoboken, NJ. All rights reserved. This material is protected under all copyright laws
as they currently exist. No portion of this material may be reproduced, in any form or by any means, without permission in writing from the publisher.

*2–88.

Express each force as a Cartesian vector, and then determine z


the magnitude and coordinate direction angles of the
resultant force. C

13 12 F1  80 lb
5

2.5 ft
O y
4 ft A
SOLUTION
F2  50 lb
12
rAC = e - 2.5 i - 4 j + (2.5) k f ft
5 x
rAC 6 ft
F1 = 80 lb a b = - 26.20 i - 41.93 j + 62.89 k
rAC
= { - 26.2 i - 41.9 j + 62.9 k} lb  Ans.
2 ft
rAB = {2 i - 4 j - 6 k} ft B

rAB
F2 = 50 lb a b = 13.36 i - 26.73 j - 40.09 k
rAB
= {13.4 i - 26.7 j - 40.1 k} lb  Ans.
FR = F1 + F2
= - 12.84 i - 68.65 j + 22.80 k
= { -12.8 i - 68.7 j + 22.8 k } lb
FR = 2( -12.84)2 ( - 68.65)2 + (22.80)2 = 73.47 = 73.5 lb Ans.

- 12.84
a = cos - 1 a b = 100° Ans.
73.47

- 68.65
b = cos - 1 a b = 159° Ans.
73.47

22.80
g = cos - 1 a b = 71.9° Ans.
73.47

Ans:
F1 = { - 26.2 i - 41.9 j + 62.9 k} lb
F2 = {13.4 i - 26.7 j - 40.1 k} lb
FR = 73.5 lb
 a = 100°
 b = 159°
 g = 71.9°

109
© 2022 by R. C. Hibbeler. Published by Pearson Education, Inc., Hoboken, NJ. All rights reserved. This material is protected under all copyright laws
as they currently exist. No portion of this material may be reproduced, in any form or by any means, without permission in writing from the publisher.

2–89.
z
The positions of point A on the flag pole and point B on the
light have been measured relative to the electronic distance
meter at O. Determine the distance between A and B.
A

232 ft

308
B
138 ft
O 458
y
508

208
SOLUTION
Position Vector:
x
rA = 232( - cos 30° sin 45°i + cos 30° cos 45°j + sin 30° k)
= { - 142.070i + 142.070j + 116k} ft

rB = 138 (cos 50° cos 20°i - cos 50° sin 20°j + sin 50°) k
= {83.355i - 30.339j + 105.714k} ft

rAB = rA - rB
= ( - 142.070i + 142.070j + 116k) - (83.355i - 30.339j + 105.714k)
= { - 225.425i + 172.409j + 10.286k} ft

Magnitude:

rAB = 2( - 225.425)2 + 172.4092 + 10.2862 = 284 ft Ans.

Ans:
rAB = 284 ft

110
© 2022 by R. C. Hibbeler. Published by Pearson Education, Inc., Hoboken, NJ. All rights reserved. This material is protected under all copyright laws
as they currently exist. No portion of this material may be reproduced, in any form or by any means, without permission in writing from the publisher.

2–90.

The 8-m-long cable is anchored to the ground at A. z


If x = 4 m and y = 2 m, determine the coordinate z to the
highest point of attachment along the column.

SOLUTION
z
r = {4i + 2j + zk} m
r = 2(4)2 + (2)2 + (z)2 = 8
z = 6.63 m Ans. y
A x
x y

Ans:
6.63 m

111
© 2022 by R. C. Hibbeler. Published by Pearson Education, Inc., Hoboken, NJ. All rights reserved. This material is protected under all copyright laws
as they currently exist. No portion of this material may be reproduced, in any form or by any means, without permission in writing from the publisher.

2–91.

The 8-m-long cable is anchored to the ground at A. z


If z = 5 m, determine the location + x, + y of the support at
A. Choose a value such that x = y.

SOLUTION z
r = {xi + yj + 5k} m
r = 2(x)2 + (y)2 + (5)2 = 8
y
x = y, thus
A x
2x2 = 82 - 52 x y

x = y = 4.42 m Ans.

Ans:
 4.42 m

112
© 2022 by R. C. Hibbeler. Published by Pearson Education, Inc., Hoboken, NJ. All rights reserved. This material is protected under all copyright laws
as they currently exist. No portion of this material may be reproduced, in any form or by any means, without permission in writing from the publisher.

*2–92.

Determine the magnitude and coordinate direction angles z


of the resultant force.
0.75 m
A

FAB  250 N
FAC  400 N 3m

2m 40 2m

C B
SOLUTION
1m
Unit Vectors: The coordinates for points A, B and C are (0, - 0.75, 3) m,
B(2 cos 40°, 2 sin 40°, 0) m and C(2, -1, 0) m respectively.
x
rAB (2 cos 40° - 0)i + [2 sin 40° - ( -0.75)]j + (0 - 3)k
uAB = =
rAB 2(2 cos 40° - 0)2 + [2 sin 40° - ( -0.75)]2 + (0 - 3)2

= 0.3893i + 0.5172j - 0.7622k

rAC (2 - 0)i + [ - 1 - ( -0.75)]j + (0 - 3)k


uAC = =
rAC 2(2 - 0)2 + [ - 1 - ( - 0.75)]2 + (0 - 3)2

= 0.5534i - 0.0692j - 0.8301k

Force Vectors:
FAB = FAB uAB = 250 (0.3893i + 0.5172j - 0.7622k)

= {97.32i + 129.30j - 190.56k} N

= {97.3i + 129j - 191k} N  Ans.

FAC = FAC uAC = 400 (0.5534i - 0.06917j - 0.8301k)

= {221.35i - 27.67j - 332.02k} N

= {221i - 27.7j - 332k} N  Ans.

Resultant Force:
FR = FAB + FAC

= {97.32i + 129.30j - 190.56k} + {221.35i - 27.67j - 332.02k}

= {318.67i + 101.63j - 522.58 k} N

The magnitude of FR is

FR = 2(FR)2x + (FR)2y + (FR)2z = 2318.672 + 101.632 + ( - 522.58)2

= 620.46 N = 620 N
And its coordinate direction angles are
(FR)x 318.67
cos a = = ; a = 59.10° = 59.1° Ans.
FR 620.46

(FR)y 101.63
cos b = = ; b = 80.57° = 80.6° Ans.
FR 620.46 Ans:
 FR = 620 N
(FR)z - 522.58
cos g = = ; g = 147.38° = 147° Ans. cos a = 59.1°
FR 620.46 cos b = 80.6°
cos g = 147°

113
© 2022 by R. C. Hibbeler. Published by Pearson Education, Inc., Hoboken, NJ. All rights reserved. This material is protected under all copyright laws
as they currently exist. No portion of this material may be reproduced, in any form or by any means, without permission in writing from the publisher.

2–93.
z
The 80-ft-long cable is attached to the boom at A.
If x = 60 ft, determine the coordinates y and z to the end of
the cable B. Take z = 3y. A

SOLUTION
y
r = (60 - 0)i + (y - 0)j + (0 - 3y)k B

= 60i + yj - 3yk x

Magnitude of r:
r = 80 = 2602 + y2 + ( - 3y)2
6400 = 3600 + 10y2
y = 16.733 ft = 16.7 ft  Ans.

z = 3y = 3(16.733) = 50.2 ft Ans.

Ans:
y = 16.7 ft
z = 50.2 ft

114
© 2022 by R. C. Hibbeler. Published by Pearson Education, Inc., Hoboken, NJ. All rights reserved. This material is protected under all copyright laws
as they currently exist. No portion of this material may be reproduced, in any form or by any means, without permission in writing from the publisher.

2–94.
z
The cable is attached to the boom at A and to point B,
which has coordinates of x = 80 ft, y = 40 ft, z = 40 ft.
Determine the length of the cable and the angle between A
the positive z axis and the cable.

SOLUTION
y
rAB = (80 - 0)i + (40 - 0)j + (0 - 40)k B

= 80i + 40j - 40k x

Magnitude of rAB:
rAB = 2802 + 402 + ( - 40)2 = 97.98 ft = 98.0 ft Ans.

Unit Vector:
rAB 80i + 40j - 40k
uAB = = = 0.8165i + 0.4082j - 0.4082k
rAB 97.98
cos g = -0.4082 g = 114° Ans.

Ans:
rAB = 98.0 ft
g = 114°

115
© 2022 by R. C. Hibbeler. Published by Pearson Education, Inc., Hoboken, NJ. All rights reserved. This material is protected under all copyright laws
as they currently exist. No portion of this material may be reproduced, in any form or by any means, without permission in writing from the publisher.

2–95.

At a given instant, the position of a plane at A and a train at z


A
B are measured relative to a radar antenna at O. Determine
the distance d between A and B at this instant. To solve the
problem, formulate a position vector, directed from A to B,
and then determine its magnitude. 5 km

60°
35°
O
y
40°
SOLUTION x
25°
2 km
Position Vector: The coordinates of points A and B are
B
A( -5 cos 60° cos 35°, -5 cos 60° sin 35°, 5 sin 60°) km

= A( - 2.048, -1.434, 4.330) km

B(2 cos 25° sin 40°, 2 cos 25° cos 40°, - 2 sin 25°) km

= B(1.165, 1.389, - 0.845) km

The position vector rAB can be established from the coordinates of points A and B.

rAB = {[1.165 - ( -2.048)] i + [1.389 - ( - 1.434)] j + ( - 0.845 - 4.330) k} km

= {3.213 i + 2.822 j - 5.175 k} km

The distance between points A and B is

d = rAB = 23.2132 + 2.8222 + ( - 5.175)2 = 6.71 km Ans.

Ans:
d = 6.71 km

116
© 2022 by R. C. Hibbeler. Published by Pearson Education, Inc., Hoboken, NJ. All rights reserved. This material is protected under all copyright laws
as they currently exist. No portion of this material may be reproduced, in any form or by any means, without permission in writing from the publisher.

*2–96.

Represent each cable force as a Cartesian vector. z

C 2m
E 2m
B

FE  350 N
FC  400 N 3m
FB  400 N
SOLUTION D
2m
rC = (0 - 5)i + ( - 2 - 0)j + (3 - 0)k = { - 5i - 2j + 3k} m y
A 3m
rC = 2( -5)2 + ( -2)2 + 32 = 238 m
x
rB = (0 - 5)i + (2 - 0)j + (3 - 0)k = { - 5i + 2j + 3k} m

rB = 2( -5)2 + 22 + 32 = 238 m

rE = (0 - 2)i + (0 - 0)j + (3 - 0)k = { - 2i + 0j + 3k} m

rE = 2( -2)2 + 02 + 32 = 213 m

r
  F = Fu = F a b
r
-5i - 2j + 3k
FC = 400 a b = { - 324i - 130j + 195k} N Ans.
138
-5i + 2j + 3k
FB = 400 a b = { - 324i + 130j + 195k} N Ans.
138
-2i + 0j + 3k
FE = 350 a b = { - 194i + 291k} N Ans.
113

Ans:
FC = { - 324i - 130j + 195k} N
FB = { - 324i + 130j + 195k} N
FE = { - 194i + 291k} N

117
© 2022 by R. C. Hibbeler. Published by Pearson Education, Inc., Hoboken, NJ. All rights reserved. This material is protected under all copyright laws
as they currently exist. No portion of this material may be reproduced, in any form or by any means, without permission in writing from the publisher.

2–97.

Determine the magnitude and coordinate direction angles z


of the resultant of the two forces acting at point A.
C 2m
E 2m
B

FE  350 N
FC  400 N 3m
FB  400 N
D
SOLUTION 2m
y
 rC = (0 - 5)i + ( - 2 - 0)j + (3 - 0)k = { -5i - 2j + 3k}
A 3m
 rC = 2( - 5)2 + ( - 2)2 + (3)2 = 238 m
x
rC ( - 5i - 2j + 3k)
FC = 400 a b = 400 a b
rC 138
FC = ( -324.4428i - 129.777j + 194.666k)

rB = (0 - 5)i + (2 - 0)j + (3 - 0)k = { - 5i + 2j + 3k}

 rB = 2( - 5)2 + 22 + 32 = 238 m

rB ( - 5i + 2j + 3k)
FB = 400 a b = 400 a b
rB 138
FB = ( -324.443i + 129.777j + 194.666k)

FR = FC + FB = ( -648.89i + 389.33k)

FR = 2( -648.89)2 + (389.33)2 + 02 = 756.7242

FR = 757 N Ans.

- 648.89
a = cos-1 a b = 149.03 = 149° Ans.
756.7242
0
b = cos-1 a b = 90.0° Ans.
756.7242

389.33
g = cos-1 a b = 59.036 = 59.0° Ans.
756.7242

Ans:
FR = 757 N
a = 149°
b = 90.0°
g = 59.0°

118
© 2022 by R. C. Hibbeler. Published by Pearson Education, Inc., Hoboken, NJ. All rights reserved. This material is protected under all copyright laws
as they currently exist. No portion of this material may be reproduced, in any form or by any means, without permission in writing from the publisher.

2–98.

The force F has a magnitude of 80 lb and acts at the z


­midpoint C of the thin rod. Express the force as a Cartesian
vector. B

6 ft
C

F  80 lb
SOLUTION
rAB = ( -3i + 2j + 6k)
y
O
1
rCB = rAB = ( - 1.5i + 1j + 3k) 3 ft
2
A
rCO = rBO + rCB
2 ft
= -6k - 1.5i + 1j + 3k x
= -1.5i + 1j - 3k

rCO = 3.5
rCO
F = 80 a b = { - 34.3i + 22.9j - 68.6k} lb Ans.
rCO

Ans:
F = { -34.3i + 22.9j - 68.6k} lb

119
© 2022 by R. C. Hibbeler. Published by Pearson Education, Inc., Hoboken, NJ. All rights reserved. This material is protected under all copyright laws
as they currently exist. No portion of this material may be reproduced, in any form or by any means, without permission in writing from the publisher.

2–99.

Position vectors along the robotic arm from O to B and z


B to A are rOB = 5 100i + 300j + 400k6 mm and
rBA = 5350i + 225j - 640k6 mm, respectively. Determine
the distance from O to the grip at A.

rBA
x
A

SOLUTION
rOA = rOB + rBA y

rOA = {450i + 525j - 240k} mm

rOA = 2(450)2 + (525)2 + ( - 240)2 = 732 mm Ans.

Ans:
rOA = 732 mm

120
© 2022 by R. C. Hibbeler. Published by Pearson Education, Inc., Hoboken, NJ. All rights reserved. This material is protected under all copyright laws
as they currently exist. No portion of this material may be reproduced, in any form or by any means, without permission in writing from the publisher.

*2–100.

If rOA = 50.5i + 4j + 0.25k6 m and rOB = 50.3i + z


2j + 2k6 m, express rBA as a Cartesian vector.

rBA
x
A

SOLUTION
rOA = rOB + rBA y

0.5i + 4j + 0.25k = 0.3i + 2j + 2k + rBA

rBA = {0.2i + 2j - 1.75k} m Ans.

rAD = 2( -1)2 + 12 + ( - 0.5)2 = 1.50 m Ans.

rBD = 212 + ( - 1)2 + 0.52 = 1.50 m Ans.

rCD = 212 + 12 + ( - 1)2 = 1.73 m Ans.

Ans:
rBA = {0.2i + 2j - 1.75k} m
rAD = 1.50 m
rBD = 1.50 m
rCD = 1.73 m

121
© 2022 by R. C. Hibbeler. Published by Pearson Education, Inc., Hoboken, NJ. All rights reserved. This material is protected under all copyright laws
as they currently exist. No portion of this material may be reproduced, in any form or by any means, without permission in writing from the publisher.

2–101.

Determine the position (x, y, 0) for fixing cable BA z


so that the resultant force exerted on the pole is directed
along its axis, from B toward O. Also, what is the magnitude B
of the resultant force?
F2 5 250 N
F1 5 350 N

4m
SOLUTION F3 5 300 N

Force Vector: 2m 3m
r1 = ( -3 - 0)i + (0 - 0)j + (0 - 4)k = { - 3i + 0j - 4k} m
O
r1 = 2( -3)2 + 02 + ( - 4)2 = 5 m 3m y
r1 - 3i - 4k x
F1 = F1 a b = 350 a
  b = { - 210i - 280k} N A
r1 5 y
x
r2 = (2 - 0)i + ( - 3 - 0)j + (0 - 4)k = {2i - 3j - 4k} m
r2 = 222 + ( - 3)2 + ( - 4)2 = 229 m
r2 2i - 3j - 4k
F2 = F2 a b = 250 a
  b
r2 229
= {92.8477i - 139.2715j - 185.6953k} N

r3 = (x - 0)i + (y - 0)j + (0 - 4)k = {xi + yj - 4k} m


r3 = 2x2 + y2 + ( - 4)2 = 216 + x2 + y2
r3 xi + yj - 4k 300xi + 300yj - 1200k
F3 = F3 a b = 300 a
  b =
r3 2
216 + x + y 2
216 + x2 + y2
FR = -FR k

Resultant Force Vector:


FR = F1 + F2 + F3

-FR k = { -210i - 280k} + {92.8477i - 139.2715j - 185.6953k}


300xi + 300yj - 1200k
+
216 + x2 + y2
300x 300y
-FR k = a - 117.1523 + bi + a - 139.2715 + bj
216 + x2 + y2 216 + x2 + y2
1200
+ a - 465.6953 - bk
216 + x2 + y2

Equating i, j, k components:
300x
-117.1523 + = 0[1]
216 + x2 + y2
300y
-139.2715 + = 0[2]
216 + x2 + y2
1200
-FR = - 465.6953 - [3]
216 + x2 + y2

Solving Eqs. [1], [2] and [3] yields:


Ans:
y = 2.34 m x = 1.96 m Ans. y = 2.34 m
FR = 704 N Ans. x = 1.96 m
FR = 704 N

122
© 2022 by R. C. Hibbeler. Published by Pearson Education, Inc., Hoboken, NJ. All rights reserved. This material is protected under all copyright laws
as they currently exist. No portion of this material may be reproduced, in any form or by any means, without permission in writing from the publisher.

2–102.

Determine the magnitude and coordinate direction angles z


a, b, g of the resultant force acting on the pole. Set x = 4 m,
y = 2 m. B

F2 5 250 N
F1 5 350 N

4m
F3 5 300 N

2m 3m
SOLUTION
Force Vector: O
3m y
r1 = ( -3 - 0) i + (0 - 0) j + (0 - 4) k = { - 3i + 0j - 4k} m x
A
r1 = 2( -3)2 + 02 + ( - 4)2 = 5 m x
y

r1 - 3i - 4k
F1 = F1 a b = 350 a
  b = { - 210i - 280k} N
r1 5

r2 = (2 - 0) i + ( -3 - 0) j + (0 - 4) k = {2i - 3j - 4k} m
r2 = 222 + ( -3)2 + ( - 4)2 = 229 m
r2 2i - 3j - 4k
F2 = F2 a b = 250 a
  b
r2 229
= {92.8477i - 139.2715j - 185.6953k} N

r3 = (4 - 0) i + (2 - 0) j + (0 - 4) k = {4i + 2j - 4k} m
r3 = 242 + 22 + ( - 4)2 = 6 m
r3 - 4i + 2j - 4k
F3 = F3 a b = 300 a
  b = {200i + 100j - 200k} N
r3 6

Resultant Force Vector:


FR = F1 + F2 + F3

= { - 210i - 280k} + {92.8477i - 139.2715j - 185.6953k}

+ {200i + 100j - 200k}

= {82.8477i - 39.2715j - 665.6953k} N

FR = 282.84772 + ( - 39.2715)2 + 665.69532

= 671.98 N = 672 N  Ans.

Coordinate Direction Angles:


FR 82.8477i - 39.2715j - 665.6953k
uR = =
FR 671.98

= 0.1233i - 0.05844j - 0.9906k

cos a = 0.1233 a = 82.9° Ans.


cos b = - 0.05844 b = 93.4° Ans.
cos g = - 0.9906 g = 172° Ans. Ans:
FR = 672 N
a = 82.9°
b = 93.4°
g = 172°

123
© 2022 by R. C. Hibbeler. Published by Pearson Education, Inc., Hoboken, NJ. All rights reserved. This material is protected under all copyright laws
as they currently exist. No portion of this material may be reproduced, in any form or by any means, without permission in writing from the publisher.

2–103.

Given the three vectors A, B, and D, show that z


A # 1B + D2 = 1A # B 2 + 1A # D2.

0.2 m
0.4 m
B

0.5 m
Given the three vectors A, B, and D, show that zu D
A # (B + D) = (A # B) + (A # D).
A y

2m A 0.8 m
Given the three vectors A, B, and D, show that z
A # (B + D) = (A # B) + (A # D). B
0.6 m 2m C
SOLUTION x 2m
A
x y
Since the component of (B + D) is equal to the sum of the components of B and 2m B
D, then 2 m2 m
SOLUTION C
A # (B
+ D) = A#B
+ A#D (QED) x D F  600 N y
Since the component of (B + D) is equal to the sum of the components of B and 2m
D, then 32mm
Also, C
E
A # (B + D) = A # B + A # D (QED) D F  600 N
A # (B + D) = (A x i + A y j + A zk) # [(Bx + Dx)i + (By + Dy)j + (Bz + Dz)k]
3m
Also, = A x (Bx + Dx) + A y (By + Dy) + A z (Bz + Dz) E
A # (B + D)= =(A(ABx i ++ A
A yBj ++ A
Azk)
# + x(A+ DDx)i+ +A(BDy
B )[(B +
+D Dz+
Ayz)j ) (Bz + Dz)k]
x x y y z z x x y y

= =(AA# xB)
(Bx+ +(AD#xD)
) + A y (By + Dy) + A z (Bz + Dz) (QED)
= (A xBx + A yBy + A zBz) + (A xDx + A yDy + A zDz)

= (A # B) + (A # D) (QED)

124
© 2022 by R. C. Hibbeler. Published by Pearson Education, Inc., Hoboken, NJ. All rights reserved. This material is protected under all copyright laws
as they currently exist. No portion of this material may be reproduced, in any form or by any means, without permission in writing from the publisher.

*2–104.

Determine the angles u and f between the wire segments. z

0.2 m
0.4 m
B

0.5 m
SOLUTION u
A D y
rBA = { -0.4j - 0.5k} m ; rBA = 0.640 m
f 0.8 m
rBC = {0.8i + 0.2j - 0.5k} m ; rBC = 0.964 m

0.6 m C
rBA # rB C = 0 + (- 0.4) (0.2) + ( -0.5) ( -0.5) = 0.170 m2
x

u = cos-1 ¢ ≤ = 74.0°
0.170
Ans.
(0.640)(0.964)

rCB = { -0.8i - 0.2j + 0.5k} m ; rCB = 0.964m

rCD = { -0.8i } m ; rCD = 0.800 m

rCB # rCD = (- 0.8)( - 0.8) = 0.640 m2

f = cos-1 ¢ ≤ = 33.9°
0.640
Ans.
(0.9464) (0.800)

Ans:
u = 74.0°
f = 33.9°

125
© 2022 by R. C. Hibbeler. Published by Pearson Education, Inc., Hoboken, NJ. All rights reserved. This material is protected under all copyright laws
as they currently exist. No portion of this material may be reproduced, in any form or by any means, without permission in writing from the publisher.

2–105.

Determine the magnitude of the projected component of z


force F along the pole.
F = {–20i + 50j – 10k} lb

O
6 ft y
SOLUTION
SOLUTION
rOA = {{3i
3 i -- 2j2 j+ +6k} ft ft
6 k} 3 ft

(3i(3-i 2j
- 2+ j 6k)
+ 6 k)
Proj F
 Proj = F ## uuOA
F = 20 i++50j
OA == ((--20i j -10k)
50 - #
10k) #
2 2 + ( -2)
2(3) 2
+ 2(6)2
2(3) + ( - 2)2 + (6) 2 ft
x
Proj F = - 31.4 lb
F =

|Proj F| 31.4 lb Ans.

Ans:
|Proj F| = 31.4 lb

126
© 2022 by R. C. Hibbeler. Published by Pearson Education, Inc., Hoboken, NJ. All rights reserved. This material is protected under all copyright laws
as they currently exist. No portion of this material may be reproduced, in any form or by any means, without permission in writing from the publisher.

2–106.

Determine the components of F that act along rod AC and z


perpendicular to it. Point B is located at the midpoint of A
the rod.

4m B

4m
O F 600 N
SOLUTION x
C

3m
rAC = (-3i + 4j - 4k), rAC = 2( - 3)2 + 42 + ( -4)2 = 241 m 6m

rAC -3i + 4j + 4k 4m
rAB = = = - 1.5i + 2j - 2k D y
2 2

rAD = rAB + rBD

rBD = rAD - rAB

= (4i + 6j - 4k) - ( - 1.5i + 2j - 2k)

= {5.5i + 4j - 2k} m

rBD = 2(5.5)2 + (4)2 + (-2)2 = 7.0887 m

rBD
F = 600a b = 465.528i + 338.5659j - 169.2829k
rBD

Component of F along rAC is F| |

F # rAC (465.528i + 338.5659j - 169.2829k) # ( -3i + 4j - 4k)


F| | = =
rAC 241
F| | = 99.1408 = 99.1 N Ans.

Component of F perpendicular to rAC is F

F2 + F2|| = F2 = 6002

F2 = 6002 - 99.14082

F = 591.75 = 592 N Ans.

Ans:
F   = 99.1 N
F# = 592 N

127
© 2022 by R. C. Hibbeler. Published by Pearson Education, Inc., Hoboken, NJ. All rights reserved. This material is protected under all copyright laws
as they currently exist. No portion of this material may be reproduced, in any form or by any means, without permission in writing from the publisher.

2–107.

Determine the components of F that act along rod AC and z


perpendicular to it. Point B is located 3 m along the rod A
from end C.

4m B

4m
O F 600 N
SOLUTION x
C

3m
rCA = 3i - 4j + 4k 6m

rCA = 6.403124 4m y
D
3
rCB = (r ) = 1.40556i - 1.874085j + 1.874085k
6.403124 CA

rOB = rOC + rCB

= -3i + 4j + r CB

= -1.59444i + 2.1259j + 1.874085k

rOD = rOB + rBD

rBD = rOD - rOB = (4i + 6j) - rOB

= 5.5944i + 3.8741j - 1.874085k

rBD = 2(5.5944)2 + (3.8741)2 + ( -1.874085)2 = 7.0582

rBD
F = 600( ) = 475.568i + 329.326j - 159.311k
rBD

rAC = (- 3i + 4j - 4k), rAC = 241

Component of F along rAC is F| |

F # rAC (475.568i + 329.326j - 159.311k) # (- 3i + 4j - 4k)


F| | = =
rAC 241
F| | = 82.4351 = 82.4 N Ans.

Component of F perpendicular to rAC is F

F2 + F2|| = F2 = 6002

F2 = 6002 - 82.43512

F = 594 N Ans.

Ans:
F   = 82.4 N
F# = 594 N

128
© 2022 by R. C. Hibbeler. Published by Pearson Education, Inc., Hoboken, NJ. All rights reserved. This material is protected under all copyright laws
as they currently exist. No portion of this material may be reproduced, in any form or by any means, without permission in writing from the publisher.

*2–108.

The force F = 525i - 50j + 10k6 lb acts at the end A of z


the pipe assembly. Determine the magnitude of the
components FF 1 1and F2 which
and F2 which
act along
actthealong
axis ofAB and
AB and
perpendicular to it. F2
F
A

F1

SOLUTION 6 ft

Unit Vectors:
Vector: The
The unit
unit vector
vector along
along AB axis is
AB axis is

10 - 02i + 15 - 92j + 10 - 62k y


uAB = = - 0.5547j - 0.8321k B
210 - 022 + 15 - 922 + 10 - 622 5 ft 4 ft

Projected Component
Component of F Along AB Axis:
x
F1 = F # uAB = 125i - 50j + 10k2 # 1- 0.5547j - 0.8321k2

= 1252102 + 1 -5021 -0.55472 + 11021-0.83212

= 19.415 lb = 19.4 lb Ans.

Component of F Perpendicular
Component of to AB
Perpendicular to AB Axis:
Axis: The magnitude of force F is
F = 2252 + 1 - 5022 + 102 = 56.789 lb.

F2 = F 2 - F 21 = 56.7892 - 19.414 2 = 53.4 lb Ans.

Ans:
F1 = 19.4 lb
F2 = 53.4 lb

129
© 2022 by R. C. Hibbeler. Published by Pearson Education, Inc., Hoboken, NJ. All rights reserved. This material is protected under all copyright laws
as they currently exist. No portion of this material may be reproduced, in any form or by any means, without permission in writing from the publisher.

2–109.

A force of F = {-40k} lb acts at the end of the pipe. z


Determine the magnitudes of the components F1 and F2
which are directed along the pipe’s axis and perpendicular
to it.
O
y
3 ft
A force of F = 5 - 40k6 lb acts at the end of the pipe. z5 ft
Determine the magnitudes of the components F1 and F2
which are directed along the pipe’s axis and perpendicular 3 ft
x
to it.
O
A y
3 ft
F2 F1
5 ft
SOLUTION 3 ft
x F 5 {240 k} lb
3 i + 5 j - 3k 3i + 5j - 3k
uOA = =
2 2
23 + 5 + (- 3) 2
243 A

3i + 5j - 3k
F1 = F # uOA = (- 40 k) # a b
F2 F1
243
F  {40 k} lb
= 18.3 lb Ans.

F2 = 2F2 - F12

F2 = 2402 - 18.32 = 35.6 lb Ans.

Ans:
F1 = 18.3 lb
F2 = 35.6 lb

130
© 2022 by R. C. Hibbeler. Published by Pearson Education, Inc., Hoboken, NJ. All rights reserved. This material is protected under all copyright laws
as they currently exist. No portion of this material may be reproduced, in any form or by any means, without permission in writing from the publisher.

2–110.

Determine the magnitudes of the components of F = 600 N z


acting along and perpendicular to segment DE of the pipe
assembly.
A
2m
the magnitudes of the thecomponents of of the components of z B
Determine magnitudes z
cting along andFperpendicular to segment DE
= 600 N acting along and perpendicular to segment DE 2m
ssembly. of the pipe assembly. A x A y
2m 2m 2m
B 2m C B
ON 2m D 2m F 5 600 N
SOLUTION
: The unit vectors uEBVectors:
and uED must bevectors
determined first.
x x 3ym y
Unit The unit u and u From
EB
Fig.
must bea,determined first. From 2Fig.
ED
m
a, 2m
2m E
(0 - 4)i + (2 - 5)j r+ [0 - (0(- 2)]k C 2m C
= EB - 4)i += (2- - 5)j +- [00.5571j
0.7428i - (- 2)]k
+ 0.3714k
u = = = - 0.7428i - 0.5571j
D + 0.3714kF 600 N D
2(0 - 4)2 + (2EB- 5)r2EB+ [0 2(0
- ( -- 2
2)]4) 2
+ (2 - 5)2 + [0 - (- 2)]2
F 600 N
3m 3m
uED = -j E E
ce vector F is given bythe force vector F is given by
Thus,
600 A - 0.7428i - = FuEB+=0.3714k)
F 0.5571j = [- 445.66i
600 A - 0.7428i - 334.25j
- 0.5571j + 222.83k]
+ 0.3714k) N
= [-445.66i - 334.25j + 222.83k] N

Product: The magnitude of the


Vector Dot component
Product:
Product: of F parallel
The magnitude to segment
of the DE of F parallel to segment DE
component
ssembly is of the pipe assembly is
F # uED = A -445.66i
(FED)- 334.25j# + 222.83k B # A - j B #
paral = F uED = A -445.66i - 334.25j + 222.83k B A -j B

= ( - 445.66)(0) + ( -334.25)(
= -1) + (222.83)(0)
( - 445.66)(0) + ( -334.25)( -1) + (222.83)(0)
= 334.25 = 334 N = 334.25 = 334 N Ans. Ans.

ent of F perpendicular to segment


The component of DE of the pipe assembly
F perpendicular is DE of the pipe assembly is
to segment

er = 2F 2 - (FED)paral 2 = 26002 - 334.25


(FED
2
)per = 2F 2 - (FED=)paral Ans. 2 = 498 N
4982 N= 26002 - 334.25 Ans.

Ans:
(FED)   = 334 N
(FED) # = 498 N

131
© 2022 by R. C. Hibbeler. Published by Pearson Education, Inc., Hoboken, NJ. All rights reserved. This material is protected under all copyright laws
as they currently exist. No portion of this material may be reproduced, in any form or by any means, without permission in writing from the publisher.

2–111.

Determine the angle u between the two cables. z


C

F2  40 N
4m

u F1  70 N
A
2m
3m
y
B
2m 3m
SOLUTION
Unit Vectors: Here, the coordinates of points A, B and C are A(2, -3, 3) m, 3m
B(0, 3, 0) and C( - 2, 3, 4) m respectively. Thus, the unit vectors along AB and AC x
are
(0 - 2)i + [3 - ( -3)]j + (0 - 3)k 2 6 3
uAB = = - i + j - k
2 2
2(0 - 2) + [3 - ( - 3)] + (0 - 3) 7 7 2 7

( - 2 - 2)i + [3 - ( - 3)]j + (4 - 3)k 4 6 1


uAC = = - i + j + k
2( - 2 - 2)2 + [3 - ( - 3)]2 + (4 - 3)2 253 253 253

The Angle U Between AB and AC:

uAB # uAC = a - i + j - kb # a -
2 6 3 4 6 1
i + j + kb
7 7 7 253 253 253

2 4 6 6 3 1
= a- ba - b + a b + a - ba b
7 253 7 253 7 253
41
=
7253
Then

u = cos - 1 ( uAB # uAC ) = cos-1a


41
b = 36.43° = 36.4° Ans.
7253

Ans:
u = 36.4°

132
© 2022 by R. C. Hibbeler. Published by Pearson Education, Inc., Hoboken, NJ. All rights reserved. This material is protected under all copyright laws
as they currently exist. No portion of this material may be reproduced, in any form or by any means, without permission in writing from the publisher.

*2–112.

Determine the magnitude of the projection of the force F1 z


along cable AC.
C

F2  40 N
4m

u F1  70 N
A
2m
3m
y
B
2m 3m

3m
x

SOLUTION
Unit Vectors: Here, the coordinates of points A, B and C are A(2, -3, 3)m, B(0, 3, 0)
and C( - 2, 3, 4) m respectively. Thus, the unit vectors along AB and AC are

(0 - 2)i + [3 - ( -3)]j + (0 - 3)k 2 6 3


uAB = = - i + j - k
2
2(0 - 2) + [3 - ( - 3)] + (0- 3) 2 7 7 7
2

( - 2 - 2)i + [3 - ( -3)]j + (4 - 3)k 4 6 1


uAC = = - i + j + k
2 2 2
2( - 2 - 2) + [3 - ( - 3)] + (4 - 3) 253 253 253

Force Vector: For F1,


2 6 3
F1 = F1 uAB = 70 a - i + j - kb = { -20i + 60j - 30k} N
7 7 7
Projected Component of F1: Along AC, it is

(F1)AC = F1 # uAC = ( - 20i + 60j - 30k) # a -


4 6 1
i + j + kb
253 253 253
4 6 1
= ( - 20)a - b + 60a b + ( - 30)a b
253 253 253
= 56.32 N = 56.3 N  Ans.
The positive sign indicates that this component points in the same direction as uAC.

Ans:
(F1)AC = 56.3 N

133
© 2022 by R. C. Hibbeler. Published by Pearson Education, Inc., Hoboken, NJ. All rights reserved. This material is protected under all copyright laws
as they currently exist. No portion of this material may be reproduced, in any form or by any means, without permission in writing from the publisher.

2–113.

If F {16i 10j 14k} N, determine the magnitude of the z


projection of F along the axis of the pole and perpendicular
to it.
A

O 60°
y
SOLUTION 2m

rOA = 2 i + 4 j + a 2(4)2 + (2)2 tan 60° b k 4m


x
rOA
uOA = a b = 0.2236 i + 0.4472 j + 0.8660 k
rOA

Proj F = FuOA = (16)(0.2236) + (10)(0.4472) + ( -14)(0.8660)

|Proj F| = |- 4.0745 N| = 4.07 N Ans.

F = (16)2 + (10)2 + (- 14)2 = 23.49 N

F = (23.49)2 - (4.0745)2 = 23.1 N Ans.

Ans:
F = 23.49 N
F# = 23.1 N

134
© 2022 by R. C. Hibbeler. Published by Pearson Education, Inc., Hoboken, NJ. All rights reserved. This material is protected under all copyright laws
as they currently exist. No portion of this material may be reproduced, in any form or by any means, without permission in writing from the publisher.

2–114.
Determine
Two cablestheexert
magnitude
forces ofonthethe
projection of F1 along the
pipe. Determine the z
line of action of F .
magnitude of the 2projected component of F along the line
1
of action of F2.
F2 25 lb
60

u 60
SOLUTION x
Force Vector:
30
uF1 = cos 30° sin 30°i + cos 30° cos 30°j - sin 30°k 30 y

= 0.4330i + 0.75j - 0.5k


F1 30 lb
F1 = FRuF1 = 30(0.4330i + 0.75j - 0.5k) lb

= {12.990i + 22.5j - 15.0k} lb

Unit Vector:
Vector: One can obtain the angle a = 135° for F2 using Eq. 2–8.
cos2 a + cos2 b + cos2 g = 1, with b = 60° and g = 60°. The unit vector along the
line of action of F2 is

uF2 = cos 135°i + cos 60°j + cos 60°k = - 0.7071i + 0.5j + 0.5k

Component of F11 Along the Line of Action of F


Projected Component F22:

(F1)F2 = F1 # uF2 = (12.990i + 22.5j - 15.0k) # (- 0.7071i + 0.5j + 0.5k)

= (12.990)(- 0.7071) + (22.5)(0.5) + (- 15.0)(0.5)

= - 5.44 lb

Negative sign indicates that the projected component of (F1)F2 acts in the opposite
sense of direction to that of uF2.

The magnitude is (F1)F2 = 5.44 lb Ans.

Ans:
(F1)F2 = 5.44 lb

135
© 2022 by R. C. Hibbeler. Published by Pearson Education, Inc., Hoboken, NJ. All rights reserved. This material is protected under all copyright laws
as they currently exist. No portion of this material may be reproduced, in any form or by any means, without permission in writing from the publisher.

2–115.

Determine the angle u between the two cables attached to z


the pipe.

F2 25 lb
60
SOLUTION
Unit Vectors: u 60

uF1 = cos 30° sin 30°i + cos 30° cos 30°j - sin 30°k x

= 0.4330i + 0.75j - 0.5k 30


30 y
uF2 = cos 135°i + cos 60°j + cos 60°k

= -0.7071i + 0.5j + 0.5k F1 30 lb

The Angles
Angles Between
BetweenTwo
Two Vectors
VectorsuU:
:

uF1 # uF2 = (0.4330i + 0.75j - 0.5k) # ( -0.7071i + 0.5j + 0.5k)

= 0.4330(- 0.7071) + 0.75(0.5) + ( -0.5)(0.5)

= -0.1812

Then,

u = cos - 1 A uF1 # uF2 B = cos - 1( -0.1812) = 100° Ans.

Ans:
u = 100°

136
© 2022 by R. C. Hibbeler. Published by Pearson Education, Inc., Hoboken, NJ. All rights reserved. This material is protected under all copyright laws
as they currently exist. No portion of this material may be reproduced, in any form or by any means, without permission in writing from the publisher.

*2–116.

Determine the magnitudes of the components of the force z


acting along the axis AB of the wrench handle and
perpendicular to it. F 5 80 N

A force of F = 80 N is applied to the handle of the wrench. z


308
Determine the angle u between the tail of the force and the u
B
handle AB. F 80 N
458

A 30
u
B
300 mm 45

A
y
x 500 mm
SOLUTION 300 mm
uF = -cos 30° sin 45° i + cos 30° cos 45° j + sin 30° k
y
x 500 mm
= -0.6124 i + 0.6124 j + 0.5 k

uAB = -j

cos u = uF # uAB = ( - 0.6124 i + 0.6124 j + 0.5 k) # ( - j)

= - 0.6124

u = 128° Ans.

Ans:
u = 128°

137
© 2022 by R. C. Hibbeler. Published by Pearson Education, Inc., Hoboken, NJ. All rights reserved. This material is protected under all copyright laws
as they currently exist. No portion of this material may be reproduced, in any form or by any means, without permission in writing from the publisher.

2–117.

Determine the magnitude of the projected component of z


the 100-lb force acting along the axis BC of the pipe. A

B
3 ft
u

8 ft
6 ft
x 4 ft
D y
SOLUTION 2 ft
F 100 lb
C
rBC = 56i^ ^ 6ft
+ 4j^ - 2k

5- 6i^ ^6
+ 8j^ + 2k
F = 100
2( - 6)2 + 82 + 22

= 5- 58.83i^ ^ 6 lb
+ 78.45j^ + 19.61k

r rBC
Fp = F # uBC = F ∙
- 78.45
= = - 10.48
|rBC| 7.483

Fp = 10.5 lb  Ans.

Ans:
Fp = 10.5 lb

138
© 2022 by R. C. Hibbeler. Published by Pearson Education, Inc., Hoboken, NJ. All rights reserved. This material is protected under all copyright laws
as they currently exist. No portion of this material may be reproduced, in any form or by any means, without permission in writing from the publisher.

2–118.

Determine the angle u between pipe segments BA and BC. z


A

B
3 ft
u

8 ft
6 ft
x 4 ft
D y
SOLUTION 2 ft
F 100 lb
C
rBC = {6i + 4j - 2k} ft

uBA = { -3i} ft
rBC # rBA - 18
u = cos-1 a b = cos-1 a
|rBC| # |rBA|
b
22.45
u = 143° Ans.

Ans:
u = 143°

139
© 2022 by R. C. Hibbeler. Published by Pearson Education, Inc., Hoboken, NJ. All rights reserved. This material is protected under all copyright laws
as they currently exist. No portion of this material may be reproduced, in any form or by any means, without permission in writing from the publisher.

2–119.

Determine the angle u between the cables AB and AC. z

1m
B 1m
2m
C

D
3m
F u

SOLUTION
y
Unit Vectors: Here, the coordinates of points A, B and C are A(6, 0, 0) m, x A 6m
B(0, -1, 2) m and C(0, 1, 3) respectively. Thus, the unit vectors along
AB and AC are

(0 - 6)i + ( -1 - 0)j + (2 - 0)k 6 1 2


uAB = = - i - j + k
2 2 2
2(0 - 6) + ( - 1 - 0) + (2 - 0) 241 241 241

(0 - 6)i + (1 - 0)j + (3 - 0)k 6 1 3


uAC = = - i + j + k
2 2 2
2(0 - 6) + (1 - 0) + (3 - 0) 246 246 246

The Angle U Between AB and AC:

uAB # uAC = a - kb # a -
6 1 2 6 1 3
i - j + i + j + kb
241 241 241 246 246 246

6 6 1 1 2 3
= a- ba - b + a- ba b + a b
241 246 241 246 241 246
41
=
21886
Then

u = cos - 1 ( UAB # UAC ) = cos-1a


41
b = 19.24998° = 19.2° Ans.
21886

Ans:
u = 19.2°

140
© 2022 by R. C. Hibbeler. Published by Pearson Education, Inc., Hoboken, NJ. All rights reserved. This material is protected under all copyright laws
as they currently exist. No portion of this material may be reproduced, in any form or by any means, without permission in writing from the publisher.

*2–120.

Determine the magnitude of the projected component of the z


force F = {400i - 200j + 500k} N acting along the cable BA.
1m
B 1m
2m
C

D
3m
F u

SOLUTION
y
Unit Vector: Here, the coordinates of points A and B are A(6, 0, 0) m and x A 6m
B(0, -1, 2) m respectively. Thus the unit vector along BA is

rBA (6 - 0)i + [0 - ( - 1)]j + (0 - 2)k 6 1 2


uBA = = = i + j - k
rBA 2(6 - 0)2 + [0 - ( - 1)]2 + (0- 2)2 241 241 241

Projected Component of F: Along BA, it is

FBA = F # uBA = (400i - 200j + 500k) # a


6 1 2
i + j - kb
241 241 241
6 1 2
= 400 a b + ( - 200)a b + 500a - b
241 241 241
= 187.41 N = 187 N  Ans.
The positive sign indicates that this component points in the same direction as uBA.

Ans:
FBA = 187 N

141
© 2022 by R. C. Hibbeler. Published by Pearson Education, Inc., Hoboken, NJ. All rights reserved. This material is protected under all copyright laws
as they currently exist. No portion of this material may be reproduced, in any form or by any means, without permission in writing from the publisher.

2–121.

Determine the magnitude of the projected component of the z


force F = {400i - 200j + 500k} N acting along the cable CA.
1m
B 1m
2m
C

D
3m
F u

SOLUTION
y
Unit Vector: Here, the coordinates of points A and C are A(6, 0, 0) m and A 6m
x
C(0, 1, 3) m respectively. Thus, the unit vector along CA is
rCA (6 - 0)i + (0 - 1)j + (0 - 3)k 6 1 3
uCA = = = i - j - k
rCA 2 2
2(6 - 0) + (0 - 1) + (0 - 3) 2
246 246 246

Projected Component of F: Along CA, it is

FCA = F # uCA = (400i - 200j + 500k) # a


6 1 3
i - j - kb
246 246 246
6 1 3
= 400a b + ( - 200)a - b + 500 a - b
246 246 246
= 162.19 N = 162 N  Ans.

The positive sign indicates that this component points in the same direction as uCA.

Ans:
FCA = 162 N

142
© 2022 by R. C. Hibbeler. Published by Pearson Education, Inc., Hoboken, NJ. All rights reserved. This material is protected under all copyright laws
as they currently exist. No portion of this material may be reproduced, in any form or by any means, without permission in writing from the publisher.

2–122.

Determine the magnitudes of the components of F acting z


along and perpendicular to segment BC of the pipe
assembly.
3 ft A
4 ft
4 ft 2 ft
B
x y

SOLUTION
Unit Vector: The unit vector uCB must be determined first. From Fig. a F {30i 45j 50k} lb
4 ft
rCB (3 - 7)i + (4 - 6)j + [0 - ( - 4)]k 2 1 2
uCB = = = - i- j + k
rCB 2 2
3(3 - 7) + (4 - 6) + [0 - ( -4)] 2 3 3 3 C

Vector Dot Product:


Product: The magnitude of the projected component of F parallel to
segment BC of the pipe assembly is

(FBC)pa = F # uCB = (30i - 45j + 50k) # ¢-


2 1 2
i - j + k≤
3 3 3

2 1 2
= (30) ¢ - ≤ + ( -45) ¢ - ≤ + 50 ¢ ≤
3 3 3

= 28.33 lb = 28.3 lb Ans.

The magnitude of F is F = 330 2 + ( - 45) 2 + 50 2 = 25425 lb. Thus, the magnitude


of the component of F perpendicular to segment BC of the pipe assembly can be
determined from

(FBC)pr = 3F2 - (FBC)pa2 = 25425 - 28.332 = 68.0 lb Ans.

Ans:
( FBC )  = 28.3 lb
( FBC ) # = 68.0 lb

143
© 2022 by R. C. Hibbeler. Published by Pearson Education, Inc., Hoboken, NJ. All rights reserved. This material is protected under all copyright laws
as they currently exist. No portion of this material may be reproduced, in any form or by any means, without permission in writing from the publisher.

2–123.

Determine the magnitude of the projected component of F z


lineExpress
along AC. AC. Express this component
this component as a Cartesian
as a Cartesian vector.
vector.
3 ft A
4 ft
4 ft 2 ft
B
x y

SOLUTION
Unit Vector: The unit vector uAC must be determined first. From Fig. a F {30i 45j 50k} lb
4 ft
(7 - 0)i + (6 - 0)j + ( - 4 - 0)k
uAC = = 0.6965 i + 0.5970 j - 0.3980 k
C
3(7 - 0)2 + (6 - 0)2 + ( - 4 - 0)2

Vector Dot Product: The magnitude of the projected component of F along line AC is

FAC = F # uAC = (30i - 45j + 50k) # (0.6965i + 0.5970j - 0.3980k)

= (30)(0.6965) + (- 45)(0.5970) + 50( -0.3980)

= 25.87 lb = 25.9 lb Ans.

Thus, FAC expressed in Cartesian vector form is

FAC = FAC uAC = - 25.87(0.6965i + 0.5970j - 0.3980k)

= { - 18.0i - 15.4j + 10.3k} lb Ans.

Ans:
FAC = 25.9 lb
FAC = { - 18.0i - 15.4j + 10.3k} lb

144
© 2022 by R. C. Hibbeler. Published by Pearson Education, Inc., Hoboken, NJ. All rights reserved. This material is protected under all copyright laws
as they currently exist. No portion of this material may be reproduced, in any form or by any means, without permission in writing from the publisher.

*2–124.

Determine the angle u between the pipe segments BA and BC. z

3 ft A
4 ft
4 ft 2 ft
B
x y

SOLUTION F {30i 45j 50k} lb


Position
Position Vectors:
Vectors: The position vectors rBA and rBC must be determined first. From 4 ft
Fig. a,
C
rBA = (0 - 3)i + (0 - 4)j + (0 - 0)k = { - 3i - 4j} ft
rBC = (7 - 3)i + (6 - 4)j + (- 4 - 0)k = {4i + 2j - 4k} ft

The magnitude of rBA and rBC are

rBA = 3(- 3)2 + (- 4)2 = 5 ft


rBC = 342 + 22 + (- 4)2 = 6 ft

Product:
Vector Dot Product:

rBA # rBC = (- 3i - 4j) # (4i + 2j - 4k)


= (- 3)(4) + ( -4)(2) + 0( -4)

= - 20 ft2

Thus,
rBA # rBC -20
u = cos-1 a b = cos-1 c d = 132° Ans.
rBA rBC 5(6)

Ans:
u = 132°

145
© 2022 by R. C. Hibbeler. Published by Pearson Education, Inc., Hoboken, NJ. All rights reserved. This material is protected under all copyright laws
as they currently exist. No portion of this material may be reproduced, in any form or by any means, without permission in writing from the publisher.

2–125.

Determine the magnitudes of the projection of the force z


F = 5 - 80i + 30j + 20k6 lb in the direction of the cables
AB and AC. F

3 ft
8 ft
C

4 ft
y
SOLUTION
5 ft
( - 4 - 0)i + (3 - 0)j + (0 - 8)k - 4i + 3j - 8k B
uAC = =
2( - 4 - 0)2 + (3 - 0)2 + (0 - 8)2 289
4 ft

(0 - 5)i + [0 - ( - 4)]j + (8 - 0)k - 5i + 4j + 8k


uBA = = x
2 2 2
2(0 - 5) + [0 - ( - 4)] + (8 - 0) 2105
- 4i + 3j - 8k
FAC = F # uAC = ( -80i - 30j + 20k) # a b
289
( -80)( - 4) + 30(3) + 20( - 8)
=
289
= 26.5 lb  Ans.
- 5i + 4j + 8k
FBA = F # uBA = ( -80i + 30j + 20k) # a b
2105
( -80)( - 5) + 30(4) + 20(8)
=
2105
= 66.4 lb  Ans.

Ans:
FAC = 26.5 lb
FBA = 66.4 lb

146
© 2022 by R. C. Hibbeler. Published by Pearson Education, Inc., Hoboken, NJ. All rights reserved. This material is protected under all copyright laws
as they currently exist. No portion of this material may be reproduced, in any form or by any means, without permission in writing from the publisher.

2–126.
Determine
Determine the the magnitudes
magnitudesofofthethe
components
componentsof the force
of force z
acting parallel and perpendicular to diagonal AB of
F = 90 lb acting parallel and perpendicular to diagonal AB the
crate.
of the crate. F 90 lb
60

B
45

1 ft
SOLUTION A
y
3 ft 1.5 ft
Force and
and Unit Vector: The force vector F and unit vector uAB must be determined x
Unit Vector: C
first. From Fig. a

F = 90(- cos 60° sin 45°i + cos 60° cos 45°j + sin 60°k)

= {- 31.82i + 31.82j + 77.94k} lb


rAB (0 - 1.5)i + (3 - 0)j + (1 - 0)k 3 6 2
uAB = = = - i- j + k
rAB 3(0 - 1.5)2 + (3 - 0)2 + (1 - 0)2 7 7 7

Product: The magnitude of the projected component of F parallel to the


Vector Dot Product:
diagonal AB is

[(F)AB]pa = F # uAB = (- 31.82i + 31.82j + 77.94k) # ¢-


3 6 2
i + j + k≤
7 7 7

3 6 2
= (- 31.82) ¢ - ≤ + 31.82 ¢ ≤ + 77.94 ¢ ≤
7 7 7

= 63.18 lb = 63.2 lb Ans.

The magnitude of the component F perpendicular to the diagonal AB is

[(F)AB]pr = 3F2 - [(F)AB]pa2 = 2902 - 63.182 = 64.1 lb Ans.

Ans:
3(F )AB 4  = 63.2 lb
3(F )AB 4 # = 64.1 lb

147
© 2022 by R. C. Hibbeler. Published by Pearson Education, Inc., Hoboken, NJ. All rights reserved. This material is protected under all copyright laws
as they currently exist. No portion of this material may be reproduced, in any form or by any means, without permission in writing from the publisher.

2–127.

Determine the magnitudes


magnitudes ofofthe
theprojected
projections of the force
components of
acting along
the force F =the300
x and y axes.
N acting along the x and y axes. 30 F 300 N
z
A 30

300 mm
SOLUTION
O
Force Vector: The force vector F must be determined first. From Fig. a, 300 mm

x 300 mm y
F = -300 sin 30°sin 30°i + 300 cos 30°j + 300 sin 30°cos 30°k

= [-75i + 259.81j + 129.90k] N

Product: The magnitudes of the projected component of F along the x


Dot Product:
Vector Dot
and y axes are

Fx = F # i = A -75i + 259.81j + 129.90k B # i


= - 75(1) + 259.81(0) + 129.90(0)

= - 75 N

Fy = F # j = A - 75i + 259.81j + 129.90k B # j


= - 75(0) + 259.81(1) + 129.90(0)

= 260 N

The negative sign indicates that Fx is directed towards the negative x axis. Thus

Fx = 75 N, Fy = 260 N Ans.

Ans:
Fx = 75 N
Fy = 260 N

148
© 2022 by R. C. Hibbeler. Published by Pearson Education, Inc., Hoboken, NJ. All rights reserved. This material is protected under all copyright laws
as they currently exist. No portion of this material may be reproduced, in any form or by any means, without permission in writing from the publisher.

*2–128.

Determine the magnitude of the projection of the force


308 F 5 300 N
acting along line OA. z
A
308

300 mm
Determine the magnitude of the projected component of O
the force F = 300 N acting along line OA. 30030
mm F 300 N
z
300 mm Ay
x 30

300 mm

O
300 mm

SOLUTION x 300 mm y

Force and Unit Vector: The force vector F and unit vector uOA must be determined
first. From Fig. a

F = (- 300 sin 30° sin 30°i + 300 cos 30°j + 300 sin 30° cos 30°k)

= { - 75i + 259.81j + 129.90k} N

rOA (-0.45 - 0)i + (0.3 - 0)j + (0.2598 - 0)k


uOA = = = - 0.75i + 0.5j + 0.4330k
rOA 2( - 0.45 - 0)2 + (0.3 - 0)2 + (0.2598 - 0)2

Vector Dot Product:


Product: The magnitude of the projected component of F along line OA is

FOA = F # uOA = A - 75i + 259.81j + 129.90k B # A -0.75i + 0.5j + 0.4330k B


= ( - 75)(-0.75) + 259.81(0.5) + 129.90(0.4330)

= 242 N Ans.

Ans:
FOA = 242 N

149
© 2022 by R. C. Hibbeler. Published by Pearson Education, Inc., Hoboken, NJ. All rights reserved. This material is protected under all copyright laws
as they currently exist. No portion of this material may be reproduced, in any form or by any means, without permission in writing from the publisher.

2–129.
z
Determine the angle u between the two cables.

8 ft

10 ft
B

4 ft
10 ft
y
u FAB  12 lb
SOLUTION 6 ft

rAC # rAB 8 ft A
u = cos-1 a b x
rAC rAB
(2 i - 8 j + 10 k) # ( -6 i + 2 j + 4 k)
= cos-1 c d
122 + ( - 8)2 + 102 1( - 6)2 + 22 + 42
12
= cos-1 a b
96.99

u = 82.9° Ans.

Ans:
u = 82.9°

150
© 2022 by R. C. Hibbeler. Published by Pearson Education, Inc., Hoboken, NJ. All rights reserved. This material is protected under all copyright laws
as they currently exist. No portion of this material may be reproduced, in any form or by any means, without permission in writing from the publisher.

2–130.

Determine the projection of the force acting in the direction z


of cable AC. Express the result as a Cartesian vector.
8 ft

10 ft
B

4 ft
10 ft
y
u FAB  12 lb
SOLUTION 6 ft

rAC = {2 i - 8 j + 10 k} ft 8 ft A
x
rAB = { - 6 i + 2 j + 4 k} ft
rAB 6 2 4
FAB = 12 a b = 12 a - i + j + kb
rAB 7.483 7.483 7.483

FAB = { - 9.621 i + 3.207 j + 6.414 k} lb

2 8 10
uAC = i - j + k
12.961 12.961 12.961

Proj FAB = FAB # uAC = - 9.621 a


2 8 10
b + 3.207 a - b + 6.414 a b
12.961 12.961 12.961
= 1.4846

Proj FAB = FAB uAC

2 8 10
Proj FAB = (1.4846) c i - j + kd
12.962 12.962 12.962

Proj FAB = {0.229 i - 0.916 j + 1.15 k} lb Ans.

Ans:
Proj FAB = {0.229 i - 0.916 j + 1.15 k} lb

151

You might also like